Med-Surge Exam 4: Ch 47, 48, 49 (and other questions on the endocrine system and diabetes mellitus

Réussis tes devoirs et examens dès maintenant avec Quizwiz!

The nurse is teaching a patient with type 2 diabetes mellitus how to prevent diabetic nephropathy. Which statement made by the patient indicates that teaching has been successful? "Smokeless tobacco products decrease the risk of kidney damage." "I can help control my blood pressure by avoiding foods high in salt." "I should have yearly dilated eye examinations by an ophthalmologist." "I will avoid hypoglycemia by keeping my blood sugar above 180 mg/dL.

"I can help control my blood pressure by avoiding foods high in salt." Diabetic nephropathy is a microvascular complication associated with damage to the small blood vessels that supply the glomeruli of the kidney. Risk factors for the development of diabetic nephropathy include hypertension, genetic predisposition, smoking, and chronic hyperglycemia. Patients with diabetes are screened for nephropathy annually with a measurement of the albumin-to-creatinine ratio in urine; a serum creatinine is also needed.

The nurse instructs a patient with diabetes mellitus about a healthy eating plan. Which statement made by the patient indicates that teaching was successful? "I plan to lose 25 lb this year by following a high-protein diet." "I may have a hypoglycemic reaction if I drink alcohol on an empty stomach." "I should include more fiber in my diet than a person who does not have diabetes." "If I use an insulin pump, I will not need to limit the amount of saturated fat in my diet."

"I may have a hypoglycemic reaction if I drink alcohol on an empty stomach." Eating carbohydrates when drinking alcohol reduces the risk for alcohol-induced hypoglycemia. Intensified insulin therapy, such as the use of an insulin pump, allows considerable flexibility in food selection and can be adjusted for alterations from usual eating and exercise habits. However, saturated fat intake should still be limited to less than 7% of total daily calories. Daily fiber intake of 14 g/1000 kcal is recommended for the general population and for patients with diabetes mellitus. High-protein diets are not recommended for weight loss.

The nurse has taught a patient admitted with diabetes principles of foot care. The nurse evaluates that the patient understands the principles of foot care if the patient makes what statement? "I should only walk barefoot in nice dry weather." "I should look at the condition of my feet every day." "I am lucky my shoes fit so nice and tight because they give me firm support." "When I am allowed up out of bed, I should check the shower water with my toes."

"I should look at the condition of my feet every day." Patients with diabetes mellitus need to inspect their feet daily for broken areas that are at risk for infection and delayed wound healing. Properly fitted (not tight) shoes should be worn at all times. Water temperature should be tested with the hands first.

The nurse teaches a patient recently diagnosed with type 1 diabetes mellitus about insulin administration. Which statement by the patient requires an intervention by the nurse? "I will discard any insulin bottle that is cloudy in appearance." "The best injection site for insulin administration is in my abdomen." "I can wash the site with soap and water before insulin administration." "I may keep my insulin at room temperature (75oF) for up to 1 month."

"I will discard any insulin bottle that is cloudy in appearance." Intermediate-acting insulin and combination-premixed insulin will be cloudy in appearance. Routine hygiene such as washing with soap and rinsing with water is adequate for skin preparation for the patient during self-injections. Insulin vials that the patient is currently using may be left at room temperature for up to 4 weeks unless the room temperature is higher than 86°F (30°C) or below freezing

The nurse is teaching a patient with type 2 diabetes mellitus about exercise to help control blood glucose. The nurse knows the patient understands when the patient elicits which exercise plan? "I want to go fishing for 30 minutes each day; I will drink fluids and wear sunscreen." "I will go running each day when my blood sugar is too high to bring it back to normal." "I will plan to keep my job as a teacher because I get a lot of exercise every school day." "I will take a brisk 30-minute walk 5 days per week and do resistance training three times a week."

"I will take a brisk 30-minute walk 5 days per week and do resistance training three times a week." The best exercise plan for the person with type 2 diabetes is for 30 minutes of moderate activity 5 days per week and resistance training three times a week. Brisk walking is moderate activity. Fishing and teaching are light activity, and running is considered vigorous activity.

The nurse receives a phone call from a patient taking cyclophosphamide for treatment of non-Hodgkin's lymphoma. The patient tells the nurse that she has muscle cramps, weakness, and very little urine output. Which response by the nurse is best? "Start taking supplemental potassium, calcium, and magnesium." "Stop taking the medication now and call your health care provider." "These symptoms will decrease with continued use of the medication." "Increase your fluid intake to 3000 mL for 24 hours to improve your urine output."

"Stop taking the medication now and call your health care provider." Cyclophosphamide may cause syndrome of inappropriate antidiuretic hormone (SIADH). Medications that stimulate the release of ADH should be avoided or discontinued. Treatment may include restriction of fluids to 800 to 1000 mL/day. A loop diuretic such as furosemide (Lasix) is used to promote diuresis, and supplements of potassium, calcium, and magnesium may be needed.

The nurse is teaching a patient with acromegaly from an unresectable benign pituitary tumor about octreotide therapy. The nurse should provide further teaching if the patient makes which statement? "The provider will infuse this medication through an IV." "I will inject the medication in the subcutaneous layer of the skin." "The medication should decrease the growth hormone production to normal." "I will have my growth hormone level measured every 2 weeks for several weeks."

"The provider will infuse this medication through an IV." Drug therapy is an option for patients whose tumors are not surgically resectable. The primary drug used is octreotide, a somatostatin analog. It reduces growth hormone (GH) levels to normal in many patients. Octreotide is given by subcutaneous injection three times a week. GH levels are measured every 2 weeks to guide drug dosing, and then every 6 months until the desired response is obtained.

The patient received regular insulin 10 units subcutaneously at 8:30 PM for a blood glucose level of 253 mg/dL. The nurse plans to monitor this patient for signs of hypoglycemia at which time related to the insulin's peak action? 8:40 PM to 9:00 PM 9:00 PM to 11:30 PM 10:30 PM to 1:30 AM 12:30 AM to 8:30 AM

10:30 PM to 1:30 AM Regular insulin exerts peak action in 2 to 5 hours, making the patient most at risk for hypoglycemia between 10:30 PM and 1:30 AM. Rapid-acting insulin's onset is between 10 and 30 minutes with peak action and hypoglycemia most likely to occur between 9:00 PM and 11:30 PM. With intermediate acting insulin, hypoglycemia may occur from 12:30 AM to 8:30 AM.

What is the priority action for the nurse to take if the patient with type 2 diabetes complains of blurred vision and irritability? a. Call the physician. b. Administer insulin as ordered. c. Check the patient's blood glucose level. d. Assess for other neurologic symptoms.

6. Correct answer: c Rationale: Check blood glucose whenever hypoglycemia is suspected so that immediate action can be taken if necessary.

A patient with a severe pounding headache has been diagnosed with hypertension. However, the hypertension is not responding to traditional treatment. What should the nurse expect as the next step in determining a diagnosis for this patient? Administration of β-blocker medications Abdominal palpation to search for a tumor Administration of potassium-sparing diuretics A 24-hour urine collection for fractionated metanephrines

A 24-hour urine collection for fractionated metanephrines Pheochromocytoma should be suspected when hypertension does not respond to traditional treatment. The 24-hour urine collection for fractionated metanephrines is simple and reliable with elevated values in 95% of people with pheochromocytoma. In a patient with pheochromocytoma, an α-adrenergic receptor blocker is used preoperatively to reduce blood pressure. Abdominal palpation is avoided to avoid a sudden release of catecholamines and severe hypertension. Potassium-sparing diuretics are not needed. Most likely they would be used for hyperaldosteronism, which is another cause of hypertension.

The nurse is reviewing laboratory results for the clinic patients to be seen today. Which patient meets the diagnostic criteria for diabetes mellitus? A 48-yr-old woman with a hemoglobin A1C of 8.4% A 58-yr-old man with a fasting blood glucose of 111 mg/dL A 68-yr-old woman with a random plasma glucose of 190 mg/dL A 78-yr-old man with a 2-hour glucose tolerance plasma glucose of 184 mg/dL

A 48-yr-old woman with a hemoglobin A1C of 8.4% Criteria for a diagnosis of diabetes mellitus include a hemoglobin A1C of 6.5% or greater, fasting plasma glucose level of 126 mg/dL or greater, 2-hour plasma glucose level of 200 mg/dL or greater during an oral glucose tolerance test, or classic symptoms of hyperglycemia or hyperglycemic crisis with a random plasma glucose of 200 mg/dL or greater.

A patient is to receive methylprednisolone (Solu-Medrol) 100 mg. The label on the medication states: methylprednisolone 125 mg in 2 mL. How many milliliters will the nurse administer?

ANS: 1.6 A concentration of 125 mg in 2 mL will result in 100 mg in 1.6 mL

In which order will the nurse take these steps to prepare NPH 20 units and regular insulin 2 units using the same syringe? (Put a comma and a space between each answer choice [A, B, C, D, E]). a. Rotate NPH vial. b. Withdraw regular insulin. c. Withdraw 20 units of NPH. d. Inject 20 units of air into NPH vial. e. Inject 2 units of air into regular insulin vial.

ANS: A, D, E, B, C When mixing regular insulin with NPH, it is important to avoid contact between the regular insulin and the additives in the NPH that slow the onset, peak, and duration of activity in the longer-acting insulin.

Which nursing action can the nurse delegate to experienced unlicensed assistive personnel (UAP) who are working in the diabetic clinic? a. Measure the ankle-brachial index. b. Check for changes in skin pigmentation. c. Assess for unilateral or bilateral foot drop. d. Ask the patient about symptoms of depression.

ANS: A Checking systolic pressure at the ankle and brachial areas and calculating the ankle-brachial index is a procedure that can be done by UAP who have been trained in the procedure. The other assessments require more education and critical thinking and should be done by the registered nurse (RN).

Which finding indicates to the nurse that the current therapies are effective for a patient with acute adrenal insufficiency? a. Increasing serum sodium levels c. Decreasing serum chloride levels b. Decreasing blood glucose levels d. Increasing serum potassium levels

ANS: A Clinical manifestations of Addison's disease include hyponatremia and an increase in sodium level indicates improvement. The other values indicate that treatment has not been effective.

Which statement by the patient indicates a need for additional instruction in administering insulin? a. "I need to rotate injection sites among my arms, legs, and abdomen each day." b. "I can buy the 0.5-mL syringes because the line markings will be easier to see." c. "I do not need to aspirate the plunger to check for blood before injecting insulin." d. "I should draw up the regular insulin first, after injecting air into the NPH bottle."

ANS: A Rotating sites is no longer recommended because there is more consistent insulin absorption when the same site is used consistently. The other patient statements are accurate and indicate that no additional instruction is needed.

The nurse is caring for a 45-yr-old male patient during a water deprivation test. Which finding is most important for the nurse to communicate to the health care provider? a. The patient complains of intense thirst. b. The patient has a 5-lb (2.3-kg) weight loss. c. The patient's urine osmolality does not increase. d. The patient feels dizzy when sitting on the edge of the bed.

ANS: B A drop in the weight of more than 2 kg indicates severe dehydration, and the test should be discontinued. The other assessment data are not unusual with this test.

A 40-yr-old patient with suspected acromegaly is seen at the clinic. To assist in making the diagnosis, which question should the nurse ask? a. "Have you had a recent head injury?" b. "Do you have to wear larger shoes now?" c. "Is there a family history of acromegaly?" d. "Are you experiencing tremors or anxiety?"

ANS: B Acromegaly causes an enlargement of the hands and feet. Head injury and family history are not risk factors for acromegaly. Tremors and anxiety are not clinical manifestations of acromegaly.

Which information about a patient who is scheduled for an oral glucose tolerance test should be reported to the health care provider before starting the test? a. The patient reports having occasional orthostatic dizziness. b. The patient takes oral corticosteroids for rheumatoid arthritis. c. The patient has had a 10-lb weight gain in the last month. d. The patient drank several glasses of water an hour previously.

ANS: B Corticosteroids can affect blood glucose results. The other information will be provided to the health care provider but will not affect the test results.

Which additional information will the nurse need to consider when reviewing the laboratory results for a patient's total calcium level? a. The blood glucose c. The phosphate level b. The serum albumin d. The magnesium level

ANS: B Part of the total calcium is bound to albumin, so hypoalbuminemia can lead to misinterpretation of calcium levels. The other laboratory values will not affect total calcium interpretation.

A patient is being admitted with a diagnosis of Cushing syndrome. Which findings will the nurse expect during the assessment? a. Chronically low blood pressure c. Purplish streaks on the abdomen b. Bronzed appearance of the skin d. Decreased axillary and pubic hair

ANS: C Purplish-red striae on the abdomen are a common clinical manifestation of Cushing syndrome. Hypotension and bronzed-appearing skin are manifestations of Addison's disease. Decreased axillary and pubic hair occur with androgen deficiency.

Which intervention will the nurse include in the plan of care for a patient with syndrome of inappropriate antidiuretic hormone (SIADH)? a. Encourage fluids to 2 to 3 L/day. b. Monitor for increasing peripheral edema. c. Offer the patient hard candies to suck on. d. Keep head of bed elevated to 30 degrees.

ANS: C Sucking on hard candies decreases thirst for a patient on fluid restriction. Patients with SIADH are on fluid restrictions of 800 to 1000 mL/day. Peripheral edema is not seen with SIADH. The head of the bed is elevated no more than 10 degrees to increase left atrial filling pressure and decrease antidiuretic hormone (ADH) release.

A 44-yr-old female patient with Cushing syndrome is admitted for adrenalectomy. Which intervention by the nurse will be most helpful for the patient problem of disturbed body image related to changes in appearance? a. Reassure the patient that the physical changes are very common in patients with Cushing syndrome. b. Discuss the use of diet and exercise in controlling the weight gain associated with Cushing syndrome. c. Teach the patient that the metabolic impact of Cushing syndrome is of more importance than appearance. d. Remind the patient that most of the physical changes caused by Cushing syndrome will resolve after surgery.

ANS: D The most reassuring and accurate communication to the patient is that the physical and emotional changes caused by the Cushing syndrome will resolve after hormone levels return to normal postoperatively. Reassurance that the physical changes are expected or that there are more serious physiologic problems associated with Cushing syndrome are not therapeutic responses. The patient's physiological changes are caused by the high hormone levels, not by the patient's diet or exercise choices.

Which hormone level would the nurse expect to be elevated in response to a low serum cortisol level? Growth hormone (GH) Follicle-stimulating hormone (FSH) Thyroid-stimulating hormone (TSH) Adrenocorticotropic hormone (ACTH)

Adrenocorticotropic hormone (ACTH) A negative feedback mechanism controls cortisol secretion. The release of corticotropin-releasing hormone from the hypothalamus stimulates the secretion of ACTH by the anterior pituitary. ACTH stimulates the adrenal cortex to secrete corticosteroids.

The hypothalamus secretes releasing hormones and inhibiting hormones. What is the target tissue of these releasing hormones and inhibiting hormones? Pineal Adrenal cortex Anterior pituitary Posterior pituitary

Anterior pituitary The anterior pituitary is the target tissue of the releasing hormones (corticotropin-releasing hormone, thyrotropin-releasing hormone, growth hormone releasing factor, gonadotropin-releasing hormone, prolactin-releasing factor) and the inhibiting hormones (somatostatin, prolactin-inhibiting factor). These hormones release or inhibit other hormones that affect the thyroid, adrenal cortex, pancreas, reproductive organs, and all body cells. The pineal gland is not directly affected by the hormones from the hypothalamus. The posterior pituitary releases antidiuretic hormone in response to plasma osmolality changes and is not directly affected by the hypothalamus hormones.

The nurse is evaluating a patient diagnosed with type 2 diabetes mellitus. Which symptom reported by the patient correlates with the diagnosis? Excessive thirst Gradual weight gain Overwhelming fatigue Recurrent blurred vision

Excessive thirst The classic symptoms of diabetes are polydipsia (excessive thirst), polyuria, (excessive urine output), and polyphagia (increased hunger). Weight gain, fatigue, and blurred vision may all occur with type 2 diabetes, but are not classic manifestations.

A patient is seeking care because of a loss of 25 lb over the past 6 months during which the patient claims to have made no significant dietary changes. What potential problem should the nurse assess the patient for? Thyroid disorders Diabetes insipidus Pituitary dysfunction Parathyroid dysfunction

Thyroid disorders Hyperthyroidism is associated with weight loss. Alterations in pituitary function, such as diabetes insipidus, and parathyroid dysfunction are not commonly associated with unexplained weight loss.

A patient admitted with type 2 diabetes asks the nurse what "type 2" means. What is the most appropriate response by the nurse? "With type 2 diabetes, the body of the pancreas becomes inflamed." "With type 2 diabetes, the patient is totally dependent on an outside source of insulin." "With type 2 diabetes, insulin secretion is decreased, and insulin resistance is increased." "With type 2 diabetes, the body produces autoantibodies that destroy β-cells in the pancreas."

"With type 2 diabetes, insulin secretion is decreased, and insulin resistance is increased." In type 2 diabetes mellitus, the secretion of insulin by the pancreas is reduced and/or the cells of the body become resistant to insulin. The pancreas becomes inflamed with pancreatitis. The patient is totally dependent on exogenous insulin and may have had autoantibodies destroy the β-cells in the pancreas with type 1 diabetes mellitus.

The patient has an order to receive 45 mg of prednisone by mouth daily. Available are 10 mg tablets. How many tablets should the nurse prepare to give? _______ tablets

4.5 45 mg ÷ 10 mg = 4.5 tablets

Which patient with type 1 diabetes mellitus would be at the highest risk for developing hypoglycemic unawareness? A 58-yr-old patient with diabetic retinopathy A 73-yr-old patient who takes propranolol (Inderal) A 19-yr-old patient who is on the school track team A 24-yr-old patient with a hemoglobin A1C of 8.9%

A 73-yr-old patient who takes propranolol (Inderal) Hypoglycemic unawareness is a condition in which a person does not experience the warning signs and symptoms of hypoglycemia until the person becomes incoherent and combative or loses consciousness. Hypoglycemic awareness is related to autonomic neuropathy of diabetes that interferes with the secretion of counterregulatory hormones that produce these symptoms. Older patients and patients who use β-adrenergic blockers (e.g., propranolol) are at risk for hypoglycemic unawareness.

Which finding for a patient who has hypothyroidism and hypertension indicates that the nurse should contact the health care provider before administering levothyroxine (Synthroid)? a. Increased thyroxine (T4) level b. Blood pressure 112/62 mm Hg c. Distant and difficult to hear heart sounds d. Elevated thyroid stimulating hormone level

ANS: A An increased thyroxine level indicates the levothyroxine dose needs to be decreased. The other data are consistent with hypothyroidism and the nurse should administer the levothyroxine.

Which question during the assessment of a patient who has diabetes will help the nurse identify autonomic neuropathy? a. "Do you feel bloated after eating?" b. "Have you seen any skin changes?" c. "Do you need to increase your insulin dosage when you are stressed?" d. "Have you noticed any painful new ulcerations or sores on your feet?"

ANS: A Autonomic neuropathy can cause delayed gastric emptying, which results in a bloated feeling for the patient. The other questions are also appropriate to ask but would not help in identifying autonomic neuropathy.

Which laboratory value should the nurse review to determine whether a patient's hypothyroidism is caused by a problem with the anterior pituitary gland or with the thyroid gland? a. Thyroxine (T4) level b. Triiodothyronine (T3) level c. Thyroid-stimulating hormone (TSH) level d. Thyrotropin-releasing hormone (TRH) level

ANS: C A low TSH level indicates that the patient's hypothyroidism is caused by decreased anterior pituitary secretion of TSH. Low T3and T4 levels are not diagnostic of the primary cause of the hypothyroidism. TRH levels indicate the function of the hypothalamus.

A 28-yr-old male patient with type 1 diabetes reports how he manages his exercise and glucose control. Which behavior indicates that the nurse should implement additional teaching? a. The patient always carries hard candies when engaging in exercise. b. The patient goes for a vigorous walk when his glucose is 200 mg/dL. c. The patient has a peanut butter sandwich before going for a bicycle ride. d. The patient increases daily exercise when ketones are present in the urine.

ANS: D When the patient is ketotic, exercise may result in an increase in blood glucose level. Patients with type 1 diabetes should be taught to avoid exercise when ketosis is present. The other statements are correct.

Which assessment finding would the nurse expect in a patient who has been taking oral prednisone several weeks and is experiencing sudden withdrawal (select all that apply.)? BP 80/50 Heart rate 54 Glucose 63 mg/dL Sodium 148 mEq/L Potassium 6.3 mEq/L Temperature 101.1° F

BP 80/50 Glucose 63 mg/dL Potassium 6.3 mEq/L Temperature 101.1° F Sudden cessation of corticosteroid therapy can precipitate life-threatening adrenal insufficiency. During acute adrenal insufficiency, the patient exhibits severe manifestations of glucocorticoid and mineralocorticoid deficiencies, including hypotension, tachycardia, dehydration, hyponatremia, hyperkalemia, hypoglycemia, fever, weakness, and confusion.

The surgeon was unable to preserve the parathyroid glands during a radical thyroidectomy. Which laboratory value should the nurse diligently monitor? Sodium levels Calcium levels Potassium levels Blood glucose levels

Calcium levels The parathyroid gland plays a key role in maintaining calcium levels. The loss of the parathyroid gland does not directly influence potassium, sodium, and glucose.

A patient with type 2 diabetes has a urinary tract infection (UTI), is difficult to arouse, and has a blood glucose of 642 mg/dL. When the nurse assesses the urine, there are no ketones present. What nursing action is appropriate at this time? Routine insulin therapy and exercise Administer a different antibiotic for the UTI. Cardiac monitoring to detect potassium changes Administer IV fluids rapidly to correct dehydration.

Cardiac monitoring to detect potassium changes This patient has manifestations of hyperosmolar hyperglycemic syndrome (HHS). Cardiac monitoring will be needed because of the changes in the potassium level related to fluid and insulin therapy and the osmotic diuresis from the elevated serum glucose level. Routine insulin would not be enough, and exercise could be dangerous for this patient. Extra insulin will be needed. The type of antibiotic will not affect HHS. There will be a large amount of IV fluid administered, but it will be given slowly because this patient is older and may have cardiac or renal compromise, requiring hemodynamic monitoring to avoid fluid overload during fluid replacement.

The nurse is assisting a patient with newly diagnosed type 2 diabetes to learn dietary planning as part of the initial management of diabetes. The nurse would encourage the patient to limit intake of which foods to help reduce the percent of fat in the diet? Cheese Broccoli Chicken Oranges

Cheese Cheese is a product derived from animal sources and is higher in fat and calories than vegetables, fruit, and poultry. Excess fat in the diet is limited to help avoid macrovascular changes.

The nurse is caring for a patient recently started on levothyroxine for hypothyroidism. What information reported by the patient requires immediate action? Weight gain or weight loss Chest pain and palpitations Muscle weakness and fatigue Decreased appetite and constipation

Chest pain and palpitations Levothyroxine is used to treat hypothyroidism. With replacement, the patient can be overmedicated, causing hyperthyroidism. Any chest pain, heart palpitations, or heart rate greater than 100 beats/min experienced by a patient starting thyroid replacement should be reported immediately, and electrocardiography and serum cardiac enzyme tests should be performed.

The nurse has been teaching a patient with diabetes mellitus how to perform self-monitoring of blood glucose (SMBG). During evaluation of the patient's technique, the nurse identifies a need for additional teaching when the patient does what? Chooses a puncture site in the center of the finger pad Washes hands with soap and water to cleanse the site to be used Warms the finger before puncturing the finger to obtain a drop of blood Tells the nurse that the result of 110 mg/dL indicates good control of diabetes

Chooses a puncture site in the center of the finger pad The patient should select a site on the sides of the fingertips, not on the center of the finger pad because this area contains many nerve endings and would be unnecessarily painful. Washing hands, warming the finger, and knowing the results that indicate good control all show understanding of the teaching.

After thyroid surgery, the nurse suspects damage or removal of the parathyroid glands when the patient develops a. muscle weakness and weight loss. b. hyperthermia and severe tachycardia. c. hypertension and difficulty swallowing. d. laryngospasms and tingling in the hands and feet.

Correct answer: d Rationale: Painful tonic spasms of smooth and skeletal muscles can cause laryngospasms that may compromise breathing. These spasms may be related to tetany, which occurs if the parathyroid glands are removed or damaged during surgery, which leads to hypocalcemia.

All cells in the body are believed to have intracellular receptors for a. insulin. b. glucagon. c. growth hormone. d. thyroid hormone.

Correct answer: d Rationale: There are two types of receptors: those that are within the cell (e.g., steroid and thyroid hormone receptors) and those that are on the cell membrane (e.g., water-soluble hormone receptors). Thyroid hormone receptors are located inside the cell. Because these hormones are lipid soluble, they pass through the target cell membrane by passive diffusion and bind to receptor sites located in the cytoplasm or nucleus of the target cell.

You are caring for a patient with newly diagnosed type 1 diabetes. What information is essential to include in your patient teaching before discharge from the hospital (select all that apply)? a. Insulin administration b. Elimination of sugar from diet c. Need to reduce physical activity d. Use of a portable blood glucose monitor e. Hypoglycemia prevention, symptoms, and treatment

Correct answers: a, d, e Rationale: The nurse ensures that the patient understands the proper use of insulin. The nurse teaches the patient how to use the portable blood glucose monitor and how to recognize and treat signs and symptoms of hypoglycemia and hyperglycemia. These are referred to as "survival skills."

A patient is newly diagnosed with type 1 diabetes and reports a headache, changes in vision, and being anxious but does not have a portable blood glucose monitor present. Which action should the nurse advise her to take? Eat a piece of pizza. Drink some diet pop. Eat 15 g of simple carbohydrates. Take an extra dose of rapid-acting insulin.

Eat 15 g of simple carbohydrates. When the patient with type 1 diabetes is unsure about the meaning of the symptoms she is experiencing, she should treat herself for hypoglycemia to prevent seizures and coma from occurring. She should also be advised to check her blood glucose as soon as possible. The fat in the pizza and the diet pop would not allow the blood glucose to increase to eliminate the symptoms. The extra dose of rapid-acting insulin would further decrease her blood glucose.

The nurse is providing discharge instructions to a patient with diabetes insipidus. Which instruction regarding desmopressin acetate would be most appropriate? Expect to experience some nasal irritation while using this drug. Monitor for symptoms of hypernatremia as a side effect of this drug. Drink at least 3000 mL of water per day while taking this medication. Report any decrease in urinary elimination to the health care provider.

Expect to experience some nasal irritation while using this drug. Desmopressin acetate is used to treat diabetes insipidus by replacing the antidiuretic hormone that the patient is lacking. Diuresis will be decreased and is expected. Inhaled desmopressin can cause nasal irritation, headache, nausea, and other signs of hyponatremia, not hypernatremia. Drinking too much water or other fluids increases the risk of hyponatremia. The patient should follow the provider's directions for limiting fluids and be taught to seek medical attention they experience severe nausea; vomiting; severe headache; muscle weakness, spasms, or cramps; sudden weight gain; unusual tiredness; mental/mood changes; seizures; and slow or shallow breathing.

The nurse is caring for a patient after a parathyroidectomy. The nurse would prepare to administer IV calcium gluconate if the patient exhibits which clinical manifestations? Facial muscle spasms and laryngospasms Tingling in the hands and around the mouth Decreased muscle tone and muscle weakness Shortened QT interval on the electrocardiogram

Facial muscle spasms and laryngospasms Nursing care for a patient after a parathyroidectomy includes monitoring for a sudden decrease in serum calcium levels causing tetany, a condition of neuromuscular hyperexcitability. If tetany is severe (e.g., muscular spasms or laryngospasms develop), IV calcium gluconate should be administered. Mild tetany, characterized by unpleasant tingling of the hands and around the mouth, may be present but should decrease over time without treatment. Decreased muscle tone, muscle weakness, and shortened QT interval are clinical manifestations of hyperparathyroidism.

The patient reports feeling tired, gaining weight, and increasing cold sensitivity even though it is now summer. Which endocrine diagnostic test should be completed first? Free thyroxine (FT4) Serum growth hormone (GH) Follicle-stimulating hormone (FSH) Magnetic resonance imaging (MRI) of the head

Free thyroxine (FT4) The manifestations the patient is experiencing could be related to hypothyroidism. FT4 is a better indicator of thyroid function than total T4 and should be done to evaluate the patient for hypothyroidism. GH excess could cause thick, leathery, oily skin but does not demonstrate the other manifestations. FSH is manifest with menstrual irregularity and would be useful in distinguishing primary gonadal problems from pituitary insufficiency. MRI is the examination of choice for radiologic evaluation of the pituitary gland and the hypothalamus but would not be the first diagnostic study done to explore the basis of these manifestations.

The nurse caring for a patient hospitalized with diabetes mellitus would look for which laboratory test result to obtain information on the patient's past glucose control? Prealbumin level Urine ketone level Fasting glucose level Glycosylated hemoglobin level

Glycosylated hemoglobin level A glycosylated hemoglobin level detects the amount of glucose that is bound to red blood cells (RBCs). When circulating glucose levels are high, glucose attaches to the RBCs and remains there for the life of the blood cell, which is approximately 120 days. Thus, the test can give an indication of glycemic control over approximately 2 to 3 months. The prealbumin level is used to establish nutritional status and is unrelated to past glucose control. The urine ketone level will only show that hyperglycemia or starvation is probably currently occurring. The fasting glucose level only indicates current glucose control.

Which complication should the nurse monitor for in a patient undergoing a growth hormone stimulation test? Hypothermia Hypertension Hyperreflexia Hypoglycemia

Hypoglycemia A growth hormone stimulation test involves the administration of insulin or arginine (agent that stimulates insulin secretion). Therefore, the nurse should monitor the patient closely for hypoglycemia. Hypothermia and hypertension are not expected in response to insulin or arginine. Hyperreflexia is an autonomic complication of spinal cord injury.

The patient in the emergency department after a car accident is wearing medical identification listing Addison's disease. What should the nurse expect to be included in the care of this patient? Low-sodium diet Increased glucocorticoid replacement Limiting IV fluid replacement therapy Withholding mineralocorticoid replacement

Increased glucocorticoid replacement The patient with Addison's disease needs lifelong glucocorticoid and mineralocorticoid replacement and has an increased need with illness, injury, or stress, as this patient is experiencing. The patient with Addison's may need large volumes of IV fluid replacement and a high-sodium diet. Withholding mineralocorticoid replacement cannot be done for patients with Addison's disease.

The nurse is reviewing laboratory results for a patient with a 15-year history of type 2 diabetes. Which result reflects the expected pattern accompanying macrovascular disease as a complication of diabetes? Increased triglyceride levels Increased high-density lipoproteins (HDL) Decreased low-density lipoproteins (LDL) Decreased very-low-density lipoproteins (VLDL)

Increased triglyceride levels Macrovascular complications of diabetes include changes to large- and medium-sized blood vessels. They include cerebrovascular, cardiovascular, and peripheral vascular disease. Increased triglyceride levels are associated with these macrovascular changes. Increased HDL, decreased LDL, and decreased VLDL are positive in relation to atherosclerosis development.

A patient, admitted with diabetes mellitus, has a glucose level of 380 mg/dL and a moderate level of ketones in the urine. As the nurse assesses for signs of ketoacidosis, which respiratory pattern would the nurse expect to find? Central apnea Hypoventilation Kussmaul respirations Cheyne-Stokes respirations

Kussmaul respirations In diabetic ketoacidosis, the lungs try to compensate for the acidosis by blowing off volatile acids and carbon dioxide. This leads to a pattern of Kussmaul respirations, which are deep and nonlabored. Central apnea occurs because the brain temporarily stops sending signals to the muscles that control breathing, which is unrelated to ketoacidosis. Hypoventilation and Cheyne-Stokes respirations do not occur with ketoacidosis.

A patient with diabetes mellitus is scheduled for a fasting blood glucose level at 8:00 AM. The nurse instructs the patient to only drink water after what time? 6:00 PM on the evening before the test Midnight before the test 4:00 AM on the day of the test 7:00 AM on the day of the test

Midnight before the test Typically, a patient is ordered to be NPO for 8 hours before a fasting blood glucose level. For this reason, the patient who has a lab draw at 8:00 AM should not have any food or beverages containing any calories after midnight.

A patient with diabetes mellitus who has multiple infections every year needs a mitral valve replacement. What is the most important preoperative teaching the nurse should provide to prevent a cardiac infection postoperatively? Avoid sick people and wash hands. Obtain comprehensive dental care. Maintain hemoglobin A1C below 7%. Coughing and deep breathing with splinting

Obtain comprehensive dental care. A person with diabetes is at high risk for postoperative infections. The most important preoperative teaching to prevent a postoperative infection in the heart is to have the patient obtain comprehensive dental care because the risk of septicemia and infective endocarditis increases with poor dental health. Avoiding sick people, hand washing, maintaining hemoglobin A1C below 7%, and coughing and deep breathing with splinting would be important for any type of surgery but are not the priority for this patient with mitral valve replacement.

What is a nursing priority in the care of a patient with hypothyroidism? Patient teaching related to levothyroxine Providing a dark, low-stimulation environment Closely monitoring the patient's intake and output Initiating precautions related to radioactive iodine therapy

Patient teaching related to levothyroxine A euthyroid state is most often achieved in patients with hypothyroidism by the administration of levothyroxine. It is not necessary to closely monitor intake and output. Low stimulation and radioactive iodine therapy are used to treat hyperthyroidism.

Which assessment parameter has highest priority when caring for a patient undergoing a water deprivation test? Patient weight Oral temperature Arterial blood gases Serum glucose levels

Patient weight A patient is at risk for severe dehydration during a water deprivation test. The test should be discontinued and the patient rehydrated if the patient's weight drops more than 2 kg at any time. The other assessment parameters do not assess fluid balance.

The nurse interviews a patient with a history of type 2 diabetes mellitus, chronic bronchitis, and osteoarthritis who has a fasting blood glucose of 154 mg/dL. Which patient medication may be responsible for the elevated blood glucose level? Metformin Prednisone Insulin glargine Acetaminophen

Prednisone Prednisone is a corticosteroid that may cause glucose intolerance in susceptible patients by increasing gluconeogenesis and insulin resistance. Insulin (e.g., glargine) and metformin (an oral hypoglycemic agent) decrease blood glucose levels. Acetaminophen has a glucose-lowering effect.

The patient with an adrenal hyperplasia is returning from surgery after an adrenalectomy. The nurse should monitor the patient for what immediate postoperative complication? Vomiting Infection Thromboembolism Rapid blood pressure changes

Rapid blood pressure changes The risk of hemorrhage is increased with surgery on the adrenal glands as well as large amounts of hormones being released in the circulation, which may produce hypertension and cause fluid and electrolyte imbalances to occur for the first 24 to 48 hours after surgery. Vomiting, infection, and thromboembolism may occur postoperatively with any surgery.

The newly diagnosed patient with type 2 diabetes has been prescribed metformin. What should the nurse teach the patient to best explain how this medication works? Increases insulin production from the pancreas Slows the absorption of carbohydrate in the small intestine Reduces glucose production by the liver and enhances insulin sensitivity Increases insulin release from the pancreas, inhibits glucagon secretion, and decreases gastric emptying

Reduces glucose production by the liver and enhances insulin sensitivity Metformin is a biguanide that reduces glucose production by the liver and enhances the tissue's insulin sensitivity. Sulfonylureas and meglitinides increase insulin production from the pancreas. α-Glucosidase inhibitors slow the absorption of carbohydrate in the intestine. Glucagon-like peptide receptor agonists increase insulin synthesis and release from the pancreas, inhibit glucagon secretion, and decrease gastric emptying.

The surgeon was unable to spare a patient's parathyroid gland during a thyroidectomy. Which assessments should the nurse prioritize when providing postoperative care for this patient? White blood cell levels and signs of infection Serum calcium levels and signs of hypocalcemia Hemoglobin, hematocrit, and red blood cell levels Level of consciousness and signs of acute delirium

Serum calcium levels and signs of hypocalcemia Correct Loss of the parathyroid gland is associated with hypocalcemia. Whereas infection and anemia are not associated with loss of the parathyroid gland, cognitive changes are less pronounced than the signs and symptoms of hypocalcemia.

The nurse teaches a client with diabetes mellitus about differentiating between hypoglycemia and ketoacidosis. The client demonstrates an understanding of the teaching by stating that a form of glucose should be taken if which symptom or symptoms develop? Select all that apply. Polyuria Shakiness Palpitations Blurred vision Lightheadedness Fruity breath odor

Shakiness Palpitations Lightheadedness Shakiness, palpitations, and lightheadedness are signs/symptoms of hypoglycemia and would indicate the need for food or glucose. Polyuria, blurred vision, and a fruity breath odor are manifestations of hyperglycemia.

A patient is admitted with diabetes mellitus, malnutrition cellulitis, and a potassium level of 5.6 mEq/L. The nurse understands that what could be contributing factors for this laboratory result (select all that apply.)? The level may be increased as a result of dehydration that accompanies hyperglycemia. The level may be raised as a result of metabolic ketoacidosis caused by hyperglycemia. The level is consistent with renal insufficiency that can develop with renal nephropathy. The patient may be excreting extra sodium and retaining potassium because of malnutrition. This level demonstrates adequate treatment of the cellulitis and effective serum glucose control.

The level may be increased as a result of dehydration that accompanies hyperglycemia. The level may be raised as a result of metabolic ketoacidosis caused by hyperglycemia. The level is consistent with renal insufficiency that can develop with renal nephropathy. The additional stress of cellulitis may lead to an increase in the patient's serum glucose levels. Dehydration may cause hemoconcentration, resulting in elevated serum readings. The kidneys may have difficulty excreting potassium if renal insufficiency exists. Finally, the nurse must consider the potential for metabolic ketoacidosis because potassium will leave the cell when hydrogen enters in an attempt to compensate for a low pH. Malnutrition does not cause sodium excretion accompanied by potassium retention. Thus, it is not a contributing factor to this patient's potassium level. The elevated potassium level does not demonstrate adequate treatment of cellulitis or effective serum glucose control.

A patient's recent medical history is indicative of diabetes insipidus. The nurse would anticipate teaching the patient about which diagnostic test? Thyroid scan Fasting glucose test Oral glucose tolerance Water deprivation test

Water deprivation test A water deprivation test is used to diagnosis if polyuria is related to diabetes insipidus. Glucose tests and thyroid tests are not directly related to the diagnosis of diabetes insipidus.

The nurse is caring for a group of older patients in a long-term care setting. Which physical assessment findings would indicate possible endocrine dysfunction? a. Hypoglycemia, delirium, and incontinence b. Impaired reflexes, diarrhea, and hearing loss c. Fatigue, constipation, and mental impairment d. Hypotension, heat intolerance, and bradycardia

fatigue, constipation, and mental impairment Changes of aging often mimic clinical manifestations of endocrine disorders. Clinical manifestations of endocrine dysfunction such as fatigue, constipation, or mental impairment in the older adult are often missed because they are attributed solely to aging.

The nurse provides instructions to a client newly diagnosed with type 1 diabetes mellitus. The nurse recognizes accurate understanding of measures to prevent diabetic ketoacidosis when the client makes which statement? "I will stop taking my insulin if I'm too sick to eat." "I will decrease my insulin dose during times of illness." "I will adjust my insulin dose according to the level of glucose in my urine." "I will notify my health care provider (HCP) if my blood glucose level is higher than 250 mg/dL (14.2 mmol/L)."

"I will notify my health care provider (HCP) if my blood glucose level is higher than 250 mg/dL (14.2 mmol/L)." During illness, the client with type 1 diabetes mellitus is at increased risk of diabetic ketoacidosis, due to hyperglycemia associated with the stress response and due to a typically decreased caloric intake. As part of sick day management, the client with diabetes should monitor blood glucose levels and should notify the HCP if the level is higher than 250 mg/dL (14.2 mmol/L). Insulin should never be stopped. In fact, insulin may need to be increased during times of illness. Doses should not be adjusted without the HCP's advice and are usually adjusted on the basis of blood glucose levels, not urinary glucose readings.

The nurse is teaching a patient who is scheduled for an oral glucose tolerance test. Which statement indicates further teaching is required? "I will fast for at least 8 hours before the test." "Ingesting caffeine and smoking may alter the test results." "The results of this test will indicate my blood sugar control over the last 3 months." "Blood samples will be taken at intervals after I drink a sample of glucose solution."

"The results of this test will indicate my blood sugar control over the last 3 months." Glycosylated hemoglobin measures the amount of glucose bound to hemoglobin over the past 3 months, not an oral glucose tolerance test. An oral glucose tolerance test requires nothing by mouth for 8 to12 hours. Caffeine and smoking may influence test results. After ingesting the oral glucose sample, blood is drawn after 30-, 60-, and 120-minute intervals.

The nurse performs a physical assessment on patient with possible endocrine dysfunction. The patient's weight was 142 lb 6 months ago compared with a current weight of 125 lb. What percent weight change will the nurse document in the patient's health record? 12% weight loss 17% weight loss 25% weight loss 74% weight loss

12% weight loss 142 lb - 125 lb = 17 lb; (17/142) × 100 = 12%. Weight change (%) is calculated by dividing the current body weight change by the usual body weight and multiplying the result by 100. Weight change greater than 5% in 1 month, 7.5% in 3 months, or 10% in 6 months is considered severe.

To assist an older patient with diabetes to engage in moderate daily exercise, which action is most important for the nurse to take? a. Determine what types of activities the patient enjoys. b. Remind the patient that exercise improves self-esteem. c. Teach the patient about the effects of exercise on glucose level. d. Give the patient a list of activities that are moderate in intensity.

ANS: A Because consistency with exercise is important, assessment for the types of exercise that the patient finds enjoyable is the most important action by the nurse in ensuring adherence to an exercise program. The other actions may be helpful but are not the most important in improving compliance.

The nurse is caring for a 63-yr-old with a possible pituitary tumor who is scheduled for a computed tomography scan with contrast. Which information about the patient is important to discuss with the health care provider before the test? a. History of renal insufficiency b. Complains of chronic headache c. Recent bilateral visual field loss d. Blood glucose level of 134 mg/dL

ANS: A Because contrast media may cause acute kidney injury in patients with poor renal function, the health care provider will need to prescribe therapies such as IV fluids to prevent this complication. The other findings are consistent with the patient's diagnosis of a pituitary tumor.

Which finding by the nurse when assessing a patient with Hashimoto's thyroiditis and a goiter will require the most immediate action? a. New-onset changes in the patient's voice b. Elevation in the patient's T3 and T4 levels c. Resting apical pulse rate 112 beats/minute d. Bruit audible bilaterally over the thyroid gland

ANS: A Changes in the patient's voice indicate that the goiter is compressing the laryngeal nerve and may lead to airway compression. The other findings will also be reported but are expected with Hashimoto's thyroiditis and do not require immediate action.

The nurse providing care for a patient who has an adrenocortical adenoma causing hyperaldosteronism should a. monitor the blood pressure every 4 hours. b. elevate the patient's legs to relieve edema. c. monitor blood glucose level every 4 hours. d. order the patient a potassium-restricted diet.

ANS: A Hypertension caused by sodium retention is a common complication of hyperaldosteronism. Hyperaldosteronism does not cause an elevation in blood glucose. The patient will be hypokalemic and require potassium supplementation before surgery. Edema does not usually occur with hyperaldosteronism.

A 27-yr-old patient admitted with diabetic ketoacidosis (DKA) has a serum glucose level of 732 mg/dL and serum potassium level of 3.1 mEq/L. Which action prescribed by the health care provider should the nurse take first? a. Place the patient on a cardiac monitor. b. Administer IV potassium supplements. c. Ask the patient about home insulin doses. d. Start an insulin infusion at 0.1 units/kg/hr.

ANS: A Hypokalemia can lead to potentially fatal dysrhythmias such as ventricular tachycardia and ventricular fibrillation, which would be detected with electrocardiogram (ECG) monitoring. Because potassium must be infused over at least 1 hour, the nurse should initiate cardiac monitoring before infusion of potassium. Insulin should not be administered without cardiac monitoring because insulin infusion will further decrease potassium levels. Discussion of home insulin and possible causes can wait until the patient is stabilized.

Which patient action indicates a good understanding of the nurse's teaching about the use of an insulin pump? a. The patient programs the pump for an insulin bolus after eating. b. The patient changes the location of the insertion site every week. c. The patient takes the pump off at bedtime and starts it again each morning. d. The patient plans a diet with more calories than usual when using the pump.

ANS: A In addition to the basal rate of insulin infusion, the patient will adjust the pump to administer a bolus after each meal, with the dosage depending on the oral intake. The insertion site should be changed every 2 or 3 days. There is more flexibility in diet and exercise when an insulin pump is used, but it does not provide for consuming a higher calorie diet. The pump will deliver a basal insulin rate 24 hours a day.

1. A young adult patient is being seen in the clinic with increased secretion of the anterior pituitary hormones. The nurse would expect the laboratory test results to show a. increased urinary cortisol. c. elevated serum aldosterone levels. b. decreased serum thyroxine. d. low urinary catecholamines excretion.

ANS: A Increased secretion of adrenocorticotropic hormone (ACTH) by the anterior pituitary gland will lead to an increase in serum and urinary cortisol levels. An increase, rather than a decrease, in thyroxine level would be expected with increased secretion of thyroid-stimulating hormone (TSH) by the anterior pituitary. Aldosterone and catecholamine levels are not controlled by the anterior pituitary.

A female patient is scheduled for an oral glucose tolerance test. Which information from the patient's health history is important for the nurse to communicate to the health care provider regarding this test? a. The patient uses oral contraceptives. b. The patient runs several days a week. c. The patient has been pregnant three times. d. The patient has a family history of diabetes.

ANS: A Oral contraceptive use may falsely elevate oral glucose tolerance test (OGTT) values. Exercise and a family history of diabetes both can affect blood glucose but will not lead to misleading information from the OGTT. History of previous pregnancies may provide informational about gestational glucose tolerance but will not lead to misleading information from the OGTT.

The nurse determines that additional instruction is needed for a patient with chronic syndrome of inappropriate antidiuretic hormone (SIADH) when the patient makes which statement? a. "I need to shop for foods low in sodium and avoid adding salt to food." b. "I should weigh myself daily and report any sudden weight loss or gain." c. "I need to limit my fluid intake to no more than 1 quart of liquids a day." d. "I should eat foods high in potassium because diuretics cause potassium loss."

ANS: A Patients with SIADH are at risk for hyponatremia, and a sodium supplement may be prescribed. The other patient statements are correct and indicate successful teaching has occurred.

A patient with diabetes is starting on intensive insulin therapy. Which type of insulin will the nurse discuss using for mealtime coverage? a. Lispro (Humalog) c. Detemir (Levemir) b. Glargine (Lantus) d. NPH (Humulin N)

ANS: A Rapid- or short-acting insulin is used for mealtime coverage for patients receiving intensive insulin therapy. NPH, glargine, or detemir will be used as the basal insulin

The nurse has administered 4 oz of orange juice to an alert patient whose blood glucose was 62 mg/dL. Fifteen minutes later, the blood glucose is 67 mg/dL. Which action should the nurse take next? a. Give the patient 4 to 6 oz more orange juice. b. Administer the PRN glucagon (Glucagon) 1 mg IM. c. Have the patient eat some peanut butter with crackers. d. Notify the health care provider about the hypoglycemia.

ANS: A The "rule of 15" indicates that administration of quickly acting carbohydrates should be done two or three times for a conscious patient whose glucose remains less than 70 mg/dL before notifying the health care provider. More complex carbohydrates and fats may be used after the glucose has stabilized. Glucagon should be used if the patient's level of consciousness decreases so that oral carbohydrates can no longer be given.

A patient who has type 1 diabetes plans to swim laps for an hour daily at 1:00 PM. The clinic nurse will plan to teach the patient to a. check glucose level before, during, and after swimming. b. delay eating the noon meal until after the swimming class. c. increase the morning dose of neutral protamine Hagedorn (NPH) insulin. d. time the morning insulin injection so that the peak occurs while swimming.

ANS: A The change in exercise will affect blood glucose, and the patient will need to monitor glucose carefully to determine the need for changes in diet and insulin administration. Because exercise tends to decrease blood glucose, patients are advised to eat before exercising. Increasing the morning NPH or timing the insulin to peak during exercise may lead to hypoglycemia, especially with the increased exercise.

The nurse is assessing a 55-yr-old female patient with type 2 diabetes who has a body mass index (BMI) of 31 kg/m2.Which goal in the plan of care is most important for this patient? a. The patient will reach a glycosylated hemoglobin level of less than 7%. b. The patient will follow a diet and exercise plan that results in weight loss. c. The patient will choose a diet that distributes calories throughout the day. d. The patient will state the reasons for eliminating simple sugars in the diet.

ANS: A The complications of diabetes are related to elevated blood glucose and the most important patient outcome is the reduction of glucose to near-normal levels. A BMI of 30?9?kg/m2 or above is considered obese, so the other outcomes are appropriate but are not as high in priority.

A patient with diabetic ketoacidosis is brought to the emergency department. Which prescribed action should the nurse implement first? a. Infuse 1 L of normal saline per hour. b. Give sodium bicarbonate 50 mEq IV push. c. Administer regular insulin 10 U by IV push. d. Start a regular insulin infusion at 0.1 units/kg/hr.

ANS: A The most urgent patient problem is the hypovolemia associated with diabetic ketoacidosis (DKA), and the priority is to infuse IV fluids. The other actions can be done after the infusion of normal saline is initiated.

Which information will the nurse include in teaching a female patient who has peripheral arterial disease, type 2 diabetes, and sensory neuropathy of the feet and legs? a. Choose flat-soled leather shoes. b. Set heating pads on a low temperature. c. Use callus remover for corns or calluses. d. Soak feet in warm water for an hour each day.

ANS: A The patient is taught to avoid high heels and that leather shoes are preferred. The feet should be washed, but not soaked, in warm water daily. Heating pad use should be avoided. Commercial callus and corn removers should be avoided. The patient should see a specialist to treat these problems.

The nurse is caring for a patient admitted with diabetes insipidus (DI). Which information is most important to report to the health care provider? a. The patient is confused and lethargic. b. The patient reports a recent head injury. c. The patient has a urine output of 400 mL/hr. d. The patient's urine specific gravity is 1.003.

ANS: A The patient's confusion and lethargy may indicate hypernatremia and should be addressed quickly. In addition, patients with DI compensate for fluid losses by drinking copious amounts of fluids, but a patient who is lethargic will be unable to drink enough fluids and will become hypovolemic. A high urine output, low urine specific gravity, and history of a recent head injury are consistent with diabetes insipidus, but they do not require immediate nursing action to avoid life-threatening complications.

A patient receives aspart (NovoLog) insulin at 8:00 AM. At which time would the nurse anticipate the highest risk for hypoglycemia? a. 10:00 AM c. 2:00 PM b. 12:00 AM d. 4:0 PM

ANS: A The rapid-acting insulins peak in 1 to 3 hours. The patient is not at a high risk for hypoglycemia at the other listed times, although hypoglycemia may occur.

Which statements will the nurse include when teaching a patient who is scheduled for oral glucose tolerance testing in the outpatient clinic (select all that apply)? a. "You will need to avoid smoking before the test." b. "Exercise should be avoided until the testing is complete." c. "Several blood samples will be obtained during the testing." d. "You should follow a low-calorie diet the day before the test." e. "The test requires that you fast for at least 8 hours before testing."

ANS: A, C, E Smoking may affect the results of oral glucose tolerance tests. Blood samples are obtained at baseline and at 30, 60, and 120 minutes. Accuracy requires that the patient be fasting before the test. The patient should consume at least 1500 calories/day for 3 days before the test. The patient should be ambulatory and active for accurate test results.

A 29-yr-old woman with systemic lupus erythematosus has been prescribed 2 weeks of high-dose prednisone therapy. Which information about the prednisone is most important for the nurse to include? a. "Weigh yourself daily to monitor for weight gain." b. "The prednisone dose should be decreased gradually." c. "A weight-bearing exercise program will help minimize risk for osteoporosis." d. "Call the health care provider if you have mood changes with the prednisone."

ANS: B Acute adrenal insufficiency may occur if exogenous corticosteroids are suddenly stopped. Mood alterations and weight gain are possible adverse effects of corticosteroid use, but these are not life-threatening effects. Osteoporosis occurs when patients take corticosteroids for longer periods.

A patient is admitted with diabetes insipidus. Which action will be appropriate for the registered nurse (RN) to delegate to an experienced licensed practical/vocational nurse (LPN/LVN)? a. Titrate the infusion of 5% dextrose in water. b. Administer prescribed subcutaneous DDAVP. c. Assess the patient's overall hydration status every 8 hours. d. Teach the patient how to use desmopressin (DDAVP) nasal spray.

ANS: B Administration of medications is included in LPN/LVN education and scope of practice. Assessments, patient teaching, and titrating fluid infusions are more complex skills and should be done by the RN.

The nurse is planning postoperative care for a patient who is being admitted to the surgical unit from the recovery room after transsphenoidal resection of a pituitary tumor. Which nursing action should be included? a. Palpate extremities for edema. b. Measure urine volume every hour. c. Check hematocrit every 2 hours for 8 hours. d. Monitor continuous pulse oximetry for 24 hours.

ANS: B After pituitary surgery, the patient is at risk for diabetes insipidus caused by cerebral edema. Monitoring of urine output and urine specific gravity is essential. Hemorrhage is not a common problem. There is no need to check the hematocrit hourly. The patient is at risk for dehydration, not volume overload. The patient is not at high risk for problems with oxygenation, and continuous pulse oximetry is not needed.

A 30-yr-old patient has a new diagnosis of type 2 diabetes. The nurse will discuss the need to schedule a dilated eye examination a. every 2 years. c. when the patient is 39 years old. b. as soon as possible. d. within the first year after diagnosis.

ANS: B Because many patients have some diabetic retinopathy when they are first diagnosed with type 2 diabetes, a dilated eye examination is recommended at the time of diagnosis and annually thereafter. Patients with type 1 diabetes should have dilated eye examinations starting 5 years after they are diagnosed and then annually.

After change-of-shift report, which patient should the nurse assess first? a. A 19-yr-old patient with type 1 diabetes who has a hemoglobin A1C of 12% b. A 23-yr-old patient with type 1 diabetes who has a blood glucose of 40 mg/dL c. A 40-yr-old patient who is pregnant and whose oral glucose tolerance test is 202 mg/dL d. A 50-yr-old patient who uses exenatide (Byetta) and is complaining of acute abdominal pain

ANS: B Because the brain requires glucose to function, untreated hypoglycemia can cause unconsciousness, seizures, and death. The nurse will rapidly assess and treat the patient with low blood glucose. The other patients also have symptoms that require assessments or interventions, but they are not at immediate risk for life-threatening complications.

The nurse determines that demeclocycline is effective for a patient with syndrome of inappropriate antidiuretic hormone (SIADH) based on finding that the patient's a. weight has increased. c. peripheral edema is increased. b. urinary output is increased. d. urine specific gravity is increased.

ANS: B Demeclocycline blocks the action of antidiuretic hormone (ADH) on the renal tubules and increases urine output. An increase in weight or an increase in urine specific gravity indicates that the SIADH is not corrected. Peripheral edema does not occur with SIADH. A sudden weight gain without edema is a common clinical manifestation of this disorder.

A patient with a possible pituitary adenoma is scheduled for a computed tomography (CT) scan with contrast media. Which patient information is important for the nurse to communicate to the health care provider before the test? a. Bilateral poor peripheral vision c. Recent weight loss of 20 lb b. Allergies to iodine and shellfish d. Complaint of ongoing headaches

ANS: B Because the usual contrast media is iodine-based, the health care provider will need to know about the allergy before the CT scan. The other findings are common with any mass in the brain such as a pituitary adenoma.

The nurse is preparing to teach a 43-yr-old man who is newly diagnosed with type 2 diabetes about home management of the disease. Which action should the nurse take first? a. Ask the patient's family to participate in the diabetes education program. b. Assess the patient's perception of what it means to have diabetes mellitus. c. Demonstrate how to check glucose using capillary blood glucose monitoring. d. Discuss the need for the patient to actively participate in diabetes management.

ANS: B Before planning teaching, the nurse should assess the patient's interest in and ability to self-manage the diabetes. After assessing the patient, the other nursing actions may be appropriate, but planning needs to be individualized to each patient.

The nurse is assessing a male patient diagnosed with a pituitary tumor causing panhypopituitarism. Assessment findings consistent with panhypopituitarism include a. high blood pressure. c. elevated blood glucose. b. decreased facial hair. d. tachycardia and palpitations.

ANS: B Changes in male secondary sex characteristics such as decreased facial hair, testicular atrophy, diminished spermatogenesis, loss of libido, impotence, and decreased muscle mass are associated with decreases in follicle-stimulating hormone (FSH) and luteinizing hormone (LH). Fasting hypoglycemia and hypotension occur in panhypopituitarism as a result of decreases in adrenocorticotropic hormone (ACTH) and cortisol. Bradycardia is likely due to the decrease in thyroid-stimulating hormone (TSH) and thyroid hormones associated with panhypopituitarism.

Which patient action indicates good understanding of the nurse's teaching about administration of aspart (NovoLog) insulin? a. The patient avoids injecting the insulin into the upper abdominal area. b. The patient cleans the skin with soap and water before insulin administration. c. The patient stores the insulin in the freezer after administering the prescribed dose. d. The patient pushes the plunger down while removing the syringe from the injection site.

ANS: B Cleaning the skin with soap and water is acceptable. Insulin should not be frozen. The patient should leave the syringe in place for about 5 seconds after injection to be sure that all the insulin has been injected. The upper abdominal area is one of the preferred areas for insulin injection.

3. A patient seen in the emergency department for severe headache and acute confusion has a serum sodium level of 118 mEq/L. The nurse will anticipate the need for which diagnostic test? a. Urinary 17-ketosteroids b. Antidiuretic hormone level c. Growth hormone stimulation test d. Adrenocorticotropic hormone level

ANS: B Elevated levels of antidiuretic hormone will cause water retention and decrease serum sodium levels. The other tests would not be helpful in determining possible causes of the patient's hyponatremia.

The cardiac telemetry unit charge nurse receives status reports from other nursing units about four patients who need cardiac monitoring. Which patient should be transferred to the cardiac unit first? a. Patient with Hashimoto's thyroiditis and a heart rate of 102 b. Patient with tetany who has a new order for IV calcium chloride c. Patient with Cushing syndrome and a blood glucose of 140 mg/dL d. Patient with Addison's disease who takes hydrocortisone twice daily

ANS: B Emergency treatment of tetany requires IV administration of calcium; electrocardiographic monitoring will be required because cardiac arrest may occur if high calcium levels result from too-rapid administration. The information about the other patients indicates that they are more stable than the patient with tetany.

An unresponsive patient with type 2 diabetes is brought to the emergency department and diagnosed with hyperosmolar hyperglycemic syndrome (HHS). The nurse will anticipate the need to a. give 50% dextrose. c. initiate O2 by nasal cannula. b. insert an IV catheter. d. administer glargine (Lantus) insulin.

ANS: B HHS is initially treated with large volumes of IV fluids to correct hypovolemia. Regular insulin is administered, not a long-acting insulin. There is no indication that the patient requires O2. Dextrose solutions will increase the patient's blood glucose and would be contraindicated.

Which nursing assessment of a 70-yr-old patient is most important to make during initiation of thyroid replacement with levothyroxine (Synthroid)? a. Fluid balance c. Nutritional intake b. Apical pulse rate d. Orientation and alertness

ANS: B In older patients, initiation of levothyroxine therapy can increase myocardial oxygen demand and cause angina or dysrhythmias. The medication also is expected to improve mental status and fluid balance and will increase metabolic rate and nutritional needs, but these changes will not result in potentially life-threatening complications.

After receiving change-of-shift report about the following four patients, which patient should the nurse assess first? a. A 31-yr-old female patient with Cushing syndrome and a blood glucose level of 244 mg/dL b. A 70-yr-old female patient taking levothyroxine (Synthroid) who has an irregular pulse of 134 c. A 53-yr-old male patient who has Addison's disease and is due for a prescribed dose of hydrocortisone (Solu-Cortef). d. A 22-yr-old male patient admitted with syndrome of inappropriate antidiuretic hormone (SIADH) who has a serum sodium level of 130 mEq/L

ANS: B Initiation of thyroid replacement in older adults may cause angina and cardiac dysrhythmias. The patient's high pulse rate needs rapid investigation by the nurse to assess for and intervene with any cardiac problems. The other patients also require nursing assessment and/or actions but are not at risk for life-threatening complications.

Which question will the nurse in the endocrine clinic ask to help determine a patient's risk factors for goiter? a. "How much milk do you drink?" b. "What medications are you taking?" c. "Are your immunizations up to date?" d. "Have you had any recent neck injuries?"

ANS: B Medications that contain thyroid-inhibiting substances can cause goiter. Milk intake, neck injury, and immunization history are not risk factors for goiter.

The nurse determines a need for additional instruction when the patient with newly diagnosed type 1 diabetes says which of the following? a. "I will need a bedtime snack because I take an evening dose of NPH insulin." b. "I can choose any foods, as long as I use enough insulin to cover the calories." c. "I can have an occasional beverage with alcohol if I include it in my meal plan." d. "I will eat something at meal times to prevent hypoglycemia, even if I am not hungry."

ANS: B Most patients with type 1 diabetes need to plan diet choices very carefully. Patients who are using intensified insulin therapy have considerable flexibility in diet choices but still should restrict dietary intake of items such as fat, protein, and alcohol. The other patient statements are correct and indicate good understanding of the diet instruction.

A patient who had radical neck surgery to remove a malignant tumor developed hypoparathyroidism. The nurse should plan to teach the patient about a. bisphosphonates to reduce bone demineralization. b. calcium supplements to normalize serum calcium levels. c. increasing fluid intake to decrease risk for nephrolithiasis. d. including whole grains in the diet to prevent constipation.

ANS: B Oral calcium supplements are used to maintain the serum calcium in normal range and prevent the complications of hypocalcemia. Whole grain foods decrease calcium absorption and will not be recommended. Bisphosphonates will lower serum calcium levels further by preventing calcium from being reabsorbed from bone. Kidney stones are not a complication of hypoparathyroidism and low calcium levels.

A registered nurse (RN) is caring for a patient with a goiter and possible hyperthyroidism. Which action by the RN indicates that the charge nurse needs to provide the RN with additional teaching? a. The RN checks the blood pressure in both arms. b. The RN palpates the neck to assess thyroid size. c. The RN orders saline eye drops to lubricate the patient's bulging eyes. d. The RN lowers the thermostat to decrease the temperature in the room.

ANS: B Palpation can cause the release of thyroid hormones in a patient with an enlarged thyroid and should be avoided. The other actions by the new RN are appropriate when caring for a patient with an enlarged thyroid.

The nurse is taking a health history from a 29-yr-old pregnant patient at the first prenatal visit. The patient reports that she has no personal history of diabetes, but her mother has diabetes. Which action will the nurse plan to take? a. Teach the patient about administering regular insulin. b. Schedule the patient for a fasting blood glucose level. c. Teach about an increased risk for fetal problems with gestational diabetes. d. Schedule an oral glucose tolerance test for the twenty-fourth week of pregnancy.

ANS: B Patients at high risk for gestational diabetes should be screened for diabetes on the initial prenatal visit. An oral glucose tolerance test may also be used to check for diabetes, but it would be done before the twenty-fourth week. Teaching plans would depend on the outcome of a fasting blood glucose test and other tests.

Which action should the nurse take after a patient treated with intramuscular glucagon for hypoglycemia regains consciousness? a. Assess the patient for symptoms of hyperglycemia. b. Give the patient a snack of peanut butter and crackers. c. Have the patient drink a glass of orange juice or nonfat milk. d. Administer a continuous infusion of 5% dextrose for 24 hours.

ANS: B Rebound hypoglycemia can occur after glucagon administration, but having a meal containing complex carbohydrates plus protein and fat will help prevent hypoglycemia. Orange juice and nonfat milk will elevate blood glucose rapidly, but the cheese and crackers will stabilize blood glucose. Administration of IV glucose might be used in patients who were unable to take in nutrition orally. The patient should be assessed for symptoms of hypoglycemia after glucagon administration.

Which information will the nurse include when teaching a 50-yr-old male patient about somatropin (Genotropin)? a. The medication will be needed for 3 to 6 months. b. Inject the medication subcutaneously every day. c. Blood glucose levels may decrease when taking the medication. d. Stop taking the medication if swelling of the hands or feet occurs.

ANS: B Somatropin is injected subcutaneously on a daily basis, preferably in the evening. The patient will need to continue on somatropin for life. If swelling or other common adverse effects occur, the health care provider should be notified. Growth hormone will increase blood glucose levels.

The nurse will plan to monitor a patient diagnosed with a pheochromocytoma for a. flushing. c. bradycardia. b. headache. d. hypoglycemia.

ANS: B The classic clinical manifestations of pheochromocytoma are hypertension, tachycardia, severe headache, diaphoresis, and abdominal or chest pain. Elevated blood glucose may also occur because of sympathetic nervous system stimulation. Bradycardia and flushing would not be expected.

A 37-yr-old patient has just arrived in the postanesthesia recovery unit (PACU) after a thyroidectomy. Which information about the patient is most important to communicate to the surgeon? a. Difficult to awaken. c. Reports 7/10 incisional pain. b. Increasing neck swelling. d. Cardiac rate 112 beats/minute.

ANS: B The neck swelling may lead to respiratory difficulty, and rapid intervention is needed to prevent airway obstruction. The incisional pain should be treated but is not unusual after surgery. A heart rate of 112 beats/min is not unusual in a patient who has been hyperthyroid and has just arrived in the PACU from surgery. Sleepiness in the immediate postoperative period is expected.

The nurse admits a patient to the hospital in Addisonian crisis. Which patient statement supports the need to plan additional teaching? a. "I frequently eat at restaurants, and my food has a lot of added salt." b. "I had the flu earlier this week, so I couldn't take the hydrocortisone." c. "I always double my dose of hydrocortisone on the days that I go for a long run." d. "I take twice as much hydrocortisone in the morning dose as I do in the afternoon."

ANS: B The need for hydrocortisone replacement is increased with stressors such as illness, and the patient needs to be taught to call the health care provider because medication and IV fluids and electrolytes may need to be given. The other patient statements indicate appropriate management of the Addison's disease.

Which laboratory value reported to the nurse by the unlicensed assistive personnel (UAP) indicates an urgent need for the nurse's assessment of the patient? a. Bedtime glucose of 140 mg/dL b. Noon blood glucose of 52 mg/dL c. Fasting blood glucose of 130 mg/dL d. 2-hr postprandial glucose of 220 mg/dL

ANS: B The nurse should assess the patient with a blood glucose level of 52 mg/dL for symptoms of hypoglycemia and give the patient a carbohydrate-containing beverage such as orange juice. The other values are within an acceptable range or not immediately dangerous for a patient with diabetes.

The nurse identifies a need for additional teaching when the patient who is self-monitoring blood glucose a. washes the puncture site using warm water and soap. b. chooses a puncture site in the center of the finger pad. c. hangs the arm down for a minute before puncturing the site. d. says the result of 120 mg indicates good blood sugar control.

ANS: B The patient is taught to choose a puncture site at the side of the finger pad because there are fewer nerve endings along the side of the finger pad. The other patient actions indicate that teaching has been effective.

Which nursing action will be included in the plan of care for a patient with Graves' disease who has exophthalmos? a. Place cold packs on the eyes to relieve pain and swelling. b. Elevate the head of the patient's bed to reduce periorbital fluid. c. Apply alternating eye patches to protect the corneas from irritation. d. Teach the patient to blink every few seconds to lubricate the corneas.

ANS: B The patient should sit upright as much as possible to promote fluid drainage from the periorbital area. With exophthalmos, the patient is unable to close the eyes completely to blink. Lubrication of the eyes, rather than eye patches, will protect the eyes from developing corneal scarring. The swelling of the eye is not caused by excessive blood flow to the eye, so cold packs will not be helpful.

A patient with primary hyperparathyroidism has a serum phosphorus level of 1.7 mg/dL (0.55 mmol/L) and calcium of 14 mg/dL (3.5 mmol/L). Which nursing action should be included in the plan of care? a. Restrict the patient to bed rest. b. Encourage 4000 mL of fluids daily. c. Institute routine seizure precautions. d. Assess for positive Chvostek's sign.

ANS: B The patient with hypercalcemia is at risk for kidney stones, which may be prevented by a high fluid intake. Seizure precautions and monitoring for Chvostek's or Trousseau's sign are appropriate for hypocalcemic patients. The patient should engage in weight-bearing exercise to decrease calcium loss from bone.

A patient who had a subtotal thyroidectomy earlier today develops laryngeal stridor and a cramp in the right hand upon returning to the surgical nursing unit. Which collaborative action will the nurse anticipate next? a. Suction the patient's airway. b. Administer IV calcium gluconate. c. Plan for emergency tracheostomy. d. Prepare for endotracheal intubation.

ANS: B The patient's clinical manifestations of stridor and cramping are consistent with tetany caused by hypocalcemia resulting from damage to the parathyroid glands during surgery. Endotracheal intubation or tracheostomy may be needed if the calcium does not resolve the stridor. Suctioning will not correct the stridor.

A patient develops carpopedal spasms and tingling of the lips following a parathyroidectomy. Which action will provide the patient with rapid relief from the symptoms? a. Administer the prescribed muscle relaxant. b. Have the patient rebreathe from a paper bag. c. Start the PRN O2 at 2 L/min per cannula. d. Stretch the muscles with passive range of motion.

ANS: B The patient's symptoms suggest mild hypocalcemia. The symptoms of hypocalcemia will be temporarily reduced by having the patient breathe into a paper bag, which will raise the PaCO2 and create a more acidic pH. Applying as-needed O2 or range of motion will have no impact on the ionized calcium level. Calcium supplements will be given to normalize calcium levels quickly, but oral supplements will take time to be absorbed.

The nurse will teach a patient who is scheduled to complete a 24-hour urine collection for 17-ketosteroids to a. insert and maintain a retention catheter. b. keep the specimen refrigerated or on ice. c. drink at least 3 L of fluid during the 24 hours. d. void and save that specimen to start the collection.

ANS: B The specimen must be kept on ice or refrigerated until the collection is finished. Voided or catheterized specimens are acceptable for the test. The initial voided specimen is discarded. There is no fluid intake requirement for the 24-hour collection.

Which information will the nurse include when teaching a patient who has type 2 diabetes about glyburide ? a. Glyburide decreases glucagon secretion from the pancreas. b. Glyburide stimulates insulin production and release from the pancreas. c. Glyburide should be taken even if the morning blood glucose level is low. d. Glyburide should not be used for 48 hours after receiving IV contrast media.

ANS: B The sulfonylureas stimulate the production and release of insulin from the pancreas. If the glucose level is low, the patient should contact the health care provider before taking glyburide because hypoglycemia can occur with this class of medication. Metformin should be held for 48 hours after administration of IV contrast media, but this is not necessary for glyburide. Glucagon secretion is not affected by glyburide.

During the nurse's physical examination of a young adult, the patient's thyroid gland cannot be felt. The most appropriate action by the nurse is to a. palpate the patient's neck more deeply. b. document that the thyroid was nonpalpable. c. notify the health care provider immediately. d. teach the patient about thyroid hormone testing.

ANS: B The thyroid is frequently nonpalpable. The nurse should simply document the finding. There is no need to notify the health care provider immediately about a normal finding. There is no indication for thyroid-stimulating hormone (TSH) testing unless there is evidence of thyroid dysfunction. Deep palpation of the neck is not appropriate.

An active 32-yr-old male who has type 1 diabetes is being seen in the endocrine clinic. Which finding indicates a need for the nurse to discuss a possible a change in therapy with the health care provider? a. Hemoglobin A1C level of 6.2% b. Blood pressure of 140/88 mmHg c. Heart rate at rest of 58 beats/minute d. High density lipoprotein (HDL) level of 65 mg/dL

ANS: B To decrease the incidence of macrovascular and microvascular problems in patients with diabetes, the goal blood pressure is usually 130/80 mm Hg. An A1C less than 6.5%, a low resting heart rate (consistent with regular aerobic exercise in a young adult), and an HDL level of 65 mg/dL all indicate that the patient's diabetes and risk factors for vascular disease are well controlled.

A patient who has diabetes and reported burning foot pain at night receives a new prescription. Which information should the nurse teach the patient about amitriptyline ? a. Amitriptyline decreases the depression caused by your foot pain. b. Amitriptyline helps prevent transmission of pain impulses to the brain. c. Amitriptyline corrects some of the blood vessel changes that cause pain. d. Amitriptyline improves sleep and makes you less aware of nighttime pain.

ANS: B Tricyclic antidepressants (TCAs) decrease the transmission of pain impulses to the spinal cord and brain. TCAs also improve sleep quality and are used for depression, but that is not the major purpose for their use in diabetic neuropathy. The blood vessel changes that contribute to neuropathy are not affected by TCAs.

A patient has been newly diagnosed with type 2 diabetes mellitus. Which information about the patient will be most useful to the nurse who is helping the patient develop strategies for successful adaptation to this disease? a. Ideal weight c. Activity level b. Value system d. Visual changes

ANS: B When dealing with a patient with a chronic condition such as diabetes, identification of the patient's values and beliefs can assist the interprofessional team in choosing strategies for successful lifestyle change. The other information also will be useful but is not as important in developing an individualized plan for the necessary lifestyle changes.

A 56-yr-old patient who is disoriented and reports a headache and muscle cramps is hospitalized with possible syndrome of inappropriate antidiuretic hormone (SIADH). The nurse would expect the initial laboratory results to include a(n) a. elevated hematocrit. c. increased serum chloride. b. decreased serum sodium. d. low urine specific gravity.

ANS: B When water is retained, the serum sodium level will drop below normal, causing the clinical manifestations reported by the patient. The hematocrit will decrease because of the dilution caused by water retention. Urine will be more concentrated with a higher specific gravity. The serum chloride level will usually decrease along with the sodium level.

A patient who was admitted with myxedema coma and diagnosed with hypothyroidism is improving. Discharge is expected to occur in 2 days. Which teaching strategy is likely to result in effective patient self-management at home? a. Delay teaching until closer to discharge date. b. Provide written reminders of information taught. c. Offer multiple options for management of therapies. d. Ensure privacy for teaching by asking the family to leave.

ANS: B Written instructions will be helpful to the patient because initially the hypothyroid patient may be unable to remember to take medications and other aspects of self-care. Because the treatment regimen is somewhat complex, teaching should be initiated well before discharge. Family members or friends should be included in teaching because the hypothyroid patient is likely to forget some aspects of the treatment plan. A simpler regimen will be easier to understand until the patient is euthyroid.

To monitor for complications in a patient with type 2 diabetes, which tests will the nurse in the diabetic clinic schedule at least annually (select all that apply)? a. Chest x-ray b. Blood pressure c. Serum creatinine d. Urine for microalbuminuria e. Complete blood count (CBC) f. Monofilament testing of the foot

ANS: B, C, D, F Blood pressure, serum creatinine, urine testing for microalbuminuria, and monofilament testing of the foot are recommended at least annually to screen for possible microvascular and macrovascular complications of diabetes. Chest x-ray and CBC might be ordered if the patient with diabetes presents with symptoms of respiratory or infectious problems but are not routinely included in screening.

The nurse is caring for a patient following an adrenalectomy. The highest priority in the immediate postoperative period is to a. protect the patient's skin. c. balance fluids and electrolytes. b. monitor for signs of infection. d. prevent emotional disturbances.

ANS: C After adrenalectomy, the patient is at risk for circulatory instability caused by fluctuating hormone levels, and the focus of care is to assess and maintain fluid and electrolyte status through the use of IV fluids and corticosteroids. The other goals are also important for the patient but are not as immediately life threatening as the circulatory collapse that can occur with fluid and electrolyte disturbances.

A patient has just arrived on the unit after a thyroidectomy. Which action should the nurse take first? a. Observe the dressing for bleeding. b. Check the blood pressure and pulse. c. Assess the patient's respiratory effort. d. Support the patient's head with pillows.

ANS: C Airway obstruction is a possible complication after thyroidectomy because of swelling or bleeding at the site or tetany. The priority nursing action is to assess the airway. The other actions are also part of the standard nursing care postthyroidectomy but are not as high of a priority.

After a patient with a pituitary adenoma has had a hypophysectomy, the nurse will teach about the need for a. sodium restriction to prevent fluid retention. b. insulin to maintain normal blood glucose levels. c. oral corticosteroids to replace endogenous cortisol. d. chemotherapy to prevent malignant tumor recurrence.

ANS: C Antidiuretic hormone (ADH), cortisol, and thyroid hormone replacement will be needed for life after hypophysectomy. Without the effects of adrenocorticotropic hormone (ACTH) and cortisol, the blood glucose and serum sodium will be low unless cortisol is replaced. An adenoma is a benign tumor, and chemotherapy will not be needed.

A patient is taking spironolactone (Aldactone), a drug that blocks the action of aldosterone on the kidney, for hypertension. The nurse will monitor for a. increased serum sodium. c. elevated serum potassium. b. decreased urinary output. d. evidence of fluid overload.

ANS: C Because aldosterone increases the excretion of potassium, a medication that blocks aldosterone will tend to cause hyperkalemia. Aldosterone also promotes the reabsorption of sodium and water in the renal tubules, so spironolactone will tend to cause increased urine output, a decreased or normal serum sodium level, and signs of dehydration.

After the nurse has finished teaching a patient who has a new prescription for exenatide (Byetta), which patient statement indicates that the teaching has been effective? a. "I may feel hungrier than usual when I take this medicine." b. "I will not need to worry about hypoglycemia with the Byetta." c. "I should take my daily aspirin at least an hour before the Byetta." d. "I will take the pill at the same time I eat breakfast in the morning."

ANS: C Because exenatide slows gastric emptying, oral medications should be taken at least 1 hour before the exenatide to avoid slowing absorption. Exenatide is injected and increases feelings of satiety. Hypoglycemia can occur with this medication.

4. Which question from a nurse during a patient interview will provide focused information about a possible thyroid disorder? a. "What methods do you use to help cope with stress?" b. "Have you experienced any blurring or double vision?" c. "Have you had a recent unplanned weight gain or loss?" d. "Do you have to get up at night to empty your bladder?"

ANS: C Because thyroid function affects metabolic rate, changes in weight may indicate hyperfunction or hypofunction of the thyroid gland. Nocturia, visual difficulty, and changes in stress level are associated with other endocrine disorders.

5. A patient will be scheduled in the outpatient clinic for blood cortisol testing. Which instruction will the nurse provide? a. "Avoid adding any salt to your foods for 24 hours before the test." b. "You will need to lie down for 30 minutes before the blood is drawn." c. "Come to the laboratory to have the blood drawn early in the morning." d. "Do not have anything to eat or drink before the blood test is obtained."

ANS: C Cortisol levels are usually drawn in the morning, when levels are highest. The other instructions would be given to patients who were having other endocrine testing.

Which statement by a nurse to a patient newly diagnosed with type 2 diabetes is correct? a. Insulin is not used to control blood glucose in patients with type 2 diabetes. b. Complications of type 2 diabetes are less serious than those of type 1 diabetes. c. Changes in diet and exercise may control blood glucose levels in type 2 diabetes. d. Type 2 diabetes is usually diagnosed when the patient is admitted with a hyperglycemic coma.

ANS: C For some patients with type 2 diabetes, changes in lifestyle are sufficient to achieve blood glucose control. Insulin is frequently used for type 2 diabetes, complications are equally severe as for type 1 diabetes, and type 2 diabetes is usually diagnosed with routine laboratory testing or after a patient develops complications such as frequent yeast infections.

When a patient who takes metformin (Glucophage) to manage type 2 diabetes develops an allergic rash from an unknown cause, the health care provider prescribes prednisone. The nurse will anticipate that the patient may a. need a diet higher in calories while receiving prednisone. b. develop acute hypoglycemia while taking the prednisone. c. require administration of insulin while taking prednisone. d. have rashes caused by metformin-prednisone interactions.

ANS: C Glucose levels increase when patients are taking corticosteroids, and insulin may be required to control blood glucose. Hypoglycemia is not a side effect of prednisone. Rashes are not an adverse effect caused by taking metformin and prednisone simultaneously. The patient may have an increased appetite when taking prednisone but will not need a diet that is higher in calories.

The nurse reviews a patient's glycosylated hemoglobin (A1C) results to evaluate a. fasting preprandial glucose levels. b. glucose levels 2 hours after a meal. c. glucose control over the past 90 days. d. hypoglycemic episodes in the past 3 months.

ANS: C Glycosylated hemoglobin testing measures glucose control over the last 3 months. Glucose testing before/after a meal or random testing may reveal impaired glucose tolerance and indicate prediabetes, but it is not done on patients who already have a diagnosis of diabetes. There is no test to evaluate for hypoglycemic episodes in the past.

The nurse will plan to teach a patient to minimize physical and emotional stress while the patient is undergoing a. a water deprivation test. b. testing for serum T3 and T4 levels. c. a 24-hour urine test for free cortisol. d. a radioactive iodine (I-131) uptake test.

ANS: C Physical and emotional stress can affect the results of the free cortisol test. The other tests are not impacted by stress.

The health care provider suspects the Somogyi effect in a 50-yr-old patient whose 6:00 AM blood glucose is 230 mg/dL. Which action will the nurse teach the patient to take? a. Avoid snacking at bedtime. b. Increase the rapid-acting insulin dose. c. Check the blood glucose during the night d. Administer a larger dose of long-acting insulin.

ANS: C If the Somogyi effect is causing the patient's increased morning glucose level, the patient will experience hypoglycemia between 2:00 and 4:00 AM. The dose of insulin will be reduced, rather than increased. A bedtime snack is used to prevent hypoglycemic episodes during the night.

A 26-yr-old female with type 1 diabetes develops a sore throat and runny nose after caring for her sick toddler. The patient calls the clinic for advice about her symptoms and a blood glucose level of 210 mg/dL despite taking her usual glargine (Lantus) and lispro (Humalog) insulin. The nurse advises the patient to a. use only the lispro insulin until the symptoms are resolved. b. limit intake of calories until the glucose is less than 120 mg/dL. c. monitor blood glucose every 4 hours and notify the clinic if it continues to rise. d. decrease intake of carbohydrates until glycosylated hemoglobin is less than 7%.

ANS: C Infection and other stressors increase blood glucose levels and the patient will need to test blood glucose frequently, treat elevations appropriately with lispro insulin, and call the health care provider if glucose levels continue to be elevated. Discontinuing the glargine will contribute to hyperglycemia and may lead to diabetic ketoacidosis (DKA). Decreasing carbohydrate or caloric intake is not appropriate because the patient will need more calories when ill. Glycosylated hemoglobin testing is not used to evaluate short-term alterations in blood glucose.

Which action by a patient indicates that the home health nurse's teaching about glargine and regular insulin has been successful? a. The patient administers the glargine 30 minutes before each meal. b. The patient's family prefills the syringes with the mix of insulins weekly. c. The patient discards the open vials of glargine and regular insulin after 4 weeks. d. The patient draws up the regular insulin and then the glargine in the same syringe.

ANS: C Insulin can be stored at room temperature for 4 weeks. Glargine should not be mixed with other insulins or prefilled and stored. Short-acting regular insulin is administered before meals, and glargine is given once daily.

When a patient with type 2 diabetes is admitted for a cholecystectomy, which nursing action can the nurse delegate to a licensed practical/vocational nurse (LPN/LVN)? a. Communicate the blood glucose level and insulin dose to the circulating nurse in surgery. b. Discuss the reason for the use of insulin therapy during the immediate postoperative period. c. Administer the prescribed lispro (Humalog) insulin before transporting the patient to surgery. d. Plan strategies to minimize the risk for hypoglycemia or hyperglycemia during the postoperative period.

ANS: C LPN/LVN education and scope of practice includes administration of insulin. Communication about patient status with other departments, planning, and patient teaching are skills that require RN education and scope of practice.

Which information will the nurse teach a patient who has been newly diagnosed with Graves' disease? a. Exercise is contraindicated to avoid increasing metabolic rate. b. Restriction of iodine intake is needed to reduce thyroid activity. c. Antithyroid medications may take several months for full effect. d. Surgery will eventually be required to remove the thyroid gland.

ANS: C Medications used to block the synthesis of thyroid hormones may take 2 to 3 months before the full effect is seen. Large doses of iodine are used to inhibit the synthesis of thyroid hormones. Exercise using large muscle groups is encouraged to decrease the irritability and hyperactivity associated with high levels of thyroid hormones. Radioactive iodine is the most common treatment for Graves' disease, although surgery may be used.

An expected patient problem for a patient admitted to the hospital with symptoms of diabetes insipidus is a. excess fluid volume related to intake greater than output. b. impaired gas exchange related to fluid retention in lungs. c. sleep pattern disturbance related to frequent waking to void. d. risk for impaired skin integrity related to generalized edema.

ANS: C Nocturia occurs as a result of the polyuria caused by diabetes insipidus. Edema, excess fluid volume, and fluid retention are not expected.

A patient with diabetes rides a bicycle to and from work every day. Which site should the nurse teach the patient to use to administer the morning insulin? a. thigh. c. abdomen. b. buttock. d. upper arm.

ANS: C Patients should be taught not to administer insulin into a site that will be exercised because exercise will increase the rate of absorption. The thigh, buttock, and arm are all exercised by riding a bicycle.

Which information obtained by the nurse in the endocrine clinic about a patient who has been taking prednisone 40 mg daily for 3 weeks is most important to report to the health care provider? a. Patient's blood pressure is 148/94 mm Hg. b. Patient has bilateral 2+ pitting ankle edema. c. Patient stopped taking the medication 2 days ago. d. Patient has not been taking the prescribed vitamin D.

ANS: C Sudden cessation of corticosteroids after taking the medication for a week or more can lead to adrenal insufficiency, with problems such as severe hypotension and hypoglycemia. The patient will need immediate evaluation by the health care provider to prevent or treat adrenal insufficiency. The other information will also be reported but does not require rapid treatment.

A patient is admitted with tetany. Which laboratory value should the nurse plan to monitor? a. Total protein c. Ionized calcium b. Blood glucose d. Serum phosphate

ANS: C Tetany is associated with hypocalcemia. The other values would not be useful for this patient.

A few weeks after an 82-yr-old patient with a new diagnosis of type 2 diabetes has been placed on metformin (Glucophage) therapy and taught about appropriate diet and exercise, the home health nurse makes a visit. Which finding should the nurse promptly discuss with the health care provider? a. Hemoglobin A1C level is 7.9%. b. Last eye examination was 18 months ago. c. Glomerular filtration rate is decreased. d. Patient has questions about the prescribed diet.

ANS: C The decrease in renal function may indicate a need to adjust the dose of metformin or change to a different medication. In older patients, the goal for A1C may be higher in order to avoid complications associated with hypoglycemia. The nurse will plan on scheduling the patient for an eye examination and addressing the questions about diet, but the area for prompt intervention is the patient's decreased renal function.

A patient with type 2 diabetes is scheduled for a follow-up visit in the clinic several months from now. Which test will the nurse schedule to evaluate the effectiveness of treatment for the patient? a. Fasting blood glucose c. Glycosylated hemoglobin b. Oral glucose tolerance d. Urine dipstick for glucose

ANS: C The glycosylated hemoglobin (A1C) test shows the overall control of glucose over 90 to 120 days. A fasting blood level indicates only the glucose level at one time. Urine glucose testing is not an accurate reflection of blood glucose level and does not reflect the glucose over a prolonged time. Oral glucose tolerance testing is done to diagnose diabetes but is not used for monitoring glucose control after diabetes has been diagnosed.

A patient who was admitted with diabetic ketoacidosis secondary to a urinary tract infection has been weaned off an insulin drip 30 minutes ago. The patient reports feeling lightheaded and sweaty. Which action should the nurse take first? a. Infuse dextrose 50% by slow IV push. b. Administer 1 mg glucagon subcutaneously. c. Obtain a glucose reading using a finger stick. d. Have the patient drink 4 ounces of orange juice.

ANS: C The patient's clinical manifestations are consistent with hypoglycemia, and the initial action should be to check the patient's glucose with a finger stick or order a stat blood glucose. If the glucose is low, the patient should ingest a rapid-acting carbohydrate, such as orange juice. Glucagon or dextrose 50% might be given if the patient's symptoms become worse or if the patient is unconscious.

After change-of-shift report, which patient will the nurse assess first? a. A 19-yr-old patient with type 1 diabetes who was admitted with possible dawn phenomenon b. A 35-yr-old patient with type 1 diabetes whose most recent blood glucose reading was 230 mg/dL c. A 60-yr-old patient with hyperosmolar hyperglycemic syndrome who has poor skin turgor and dry oral mucosa d. A 68-yr-old patient with type 2 diabetes who has severe peripheral neuropathy and complains of burning foot pain

ANS: C The patient's diagnosis of HHS and signs of dehydration indicate that the nurse should rapidly assess for signs of shock and determine whether increased fluid infusion is needed. The other patients also need assessment and intervention but do not have life-threatening complications.

A patient screened for diabetes at a clinic has a fasting plasma glucose level of 120 mg/dL (6.7 mmol/L). The nurse will plan to teach the patient about a. self-monitoring of blood glucose. b. using low doses of regular insulin. c. lifestyle changes to lower blood glucose. d. effects of oral hypoglycemic medications.

ANS: C The patient's impaired fasting glucose indicates prediabetes, and the patient should be counseled about lifestyle changes to prevent the development of type 2 diabetes. The patient with prediabetes does not require insulin or oral hypoglycemics for glucose control and does not need to self-monitor blood glucose.

After obtaining the information shown in the accompanying figure regarding a patient with Addison's disease, which prescribed action will the nurse take first? a. Give 4 oz of fruit juice orally. b. Recheck the blood glucose level. c. Infuse 5% dextrose and 0.9% saline. d. Administer O2 therapy as needed.

ANS: C The patient's poor skin turgor, hypotension, and hyponatremia indicate an Addisonian crisis. Immediate correction of the hypovolemia and hyponatremia is needed. The other actions may also be needed but are not the initial action for the patient.

Which assessment finding for a 33-yr-old female patient admitted with Graves' disease requires the most rapid intervention by the nurse? a. Heart rate 136 beats/min c. Temperature 103.8° F (40.4° C) b. Severe bilateral exophthalmos d. Blood pressure 166/100 mm Hg

ANS: C The patient's temperature indicates that the patient may have thyrotoxic crisis and that interventions to lower the temperature are needed immediately. The other findings also require intervention but do not indicate potentially life-threatening complications.

A 62-yr-old patient with hyperthyroidism is to be treated with radioactive iodine (RAI). The nurse instructs the patient a. about radioactive precautions to take with all body secretions. b. that symptoms of hyperthyroidism should be relieved in about a week. c. that symptoms of hypothyroidism may occur as the RAI therapy takes effect. d. to discontinue the antithyroid medications taken before the radioactive therapy.

ANS: C There is a high incidence of postradiation hypothyroidism after RAI, and the patient should be monitored for symptoms of hypothyroidism. RAI has a delayed response, with the maximum effect not seen for 2 to 3 months, and the patient will continue to take antithyroid medications during this time. The therapeutic dose of radioactive iodine is low enough that no radiation safety precautions are needed.

A patient is scheduled for transsphenoidal hypophysectomy to treat a pituitary adenoma. During preoperative teaching, the nurse instructs the patient about the need to a. cough and deep breathe every 2 hours postoperatively. b. remain on bed rest for the first 48 hours after the surgery. c. avoid brushing teeth for at least 10 days after the surgery. d. be positioned flat with sandbags at the head postoperatively.

ANS: C To avoid disruption of the suture line, the patient should avoid brushing the teeth for 10 days after surgery. It is not necessary to remain on bed rest after this surgery. Coughing is discouraged because it may cause leakage of cerebrospinal fluid (CSF) from the suture line. The head of the bed should be elevated 30 degrees to reduce pressure on the sella turcica and decrease the risk for headaches.

A patient who has type 2 diabetes is being prepared for an elective coronary angiogram. Which information would the nurse anticipate might lead to rescheduling the test? a. The patient's most recent A1C was 6.5%. b. The patient's blood glucose is 128 mg/dL. c. The patient took the prescribed metformin today. d. The patient took the prescribed captopril this morning.

ANS: C To avoid lactic acidosis, metformin should be discontinued a day or 2 before the coronary angiogram and should not be used for 48 hours after IV contrast media are administered. The other patient data will also be reported but do not indicate any need to reschedule the procedure.

The nurse is assessing a 22-yr-old patient experiencing the onset of symptoms of type 1 diabetes. To which question would the nurse anticipate a positive response? a. "Are you anorexic?" c. "Have you lost weight lately?" b. "Is your urine dark colored?" d. "Do you crave sugary drinks?"

ANS: C Weight loss occurs because the body is no longer able to absorb glucose and starts to break down protein and fat for energy. The patient is thirsty but does not necessarily crave sugar-containing fluids. Increased appetite is a classic symptom of type 1 diabetes. With the classic symptom of polyuria, urine will be very dilute.

An 82-yr-old patient in a long-term care facility is newly diagnosed with hypothyroidism. The nurse will need to consult with the health care provider before administering the prescribed a. docusate (Colace). c. diazepam (Valium). b. ibuprofen (Motrin). d. cefoxitin (Mefoxin).

ANS: C Worsening of mental status and myxedema coma can be precipitated by the use of sedatives, especially in older adults. The nurse should discuss the use of diazepam with the health care provider before administration. The other medications may be given safely to the patient.

Which information is most important for the nurse to communicate rapidly to the health care provider about a patient admitted with possible syndrome of inappropriate antidiuretic hormone (SIADH)? a. The patient has a recent weight gain of 9 lb. b. The patient complains of dyspnea with activity. c. The patient has a urine specific gravity of 1.025. d. The patient has a serum sodium level of 118 mEq/L.

ANS: D A serum sodium of less than 120 mEq/L increases the risk for complications such as seizures and needs rapid correction. The other data are not unusual for a patient with SIADH and do not indicate the need for rapid action.

Which prescribed medication should the nurse expect will have rapid effects on a patient admitted to the emergency department in thyroid storm? a. Iodine c. Propylthiouracil b. Methimazole d. Propranolol (Inderal)

ANS: D B-Adrenergic blockers work rapidly to decrease the cardiovascular manifestations of thyroid storm. The other medications take days to weeks to have an impact on thyroid function.

A hospitalized diabetic patient received 38 U of NPH insulin at 7:00 AM. At 1:00 PM, the patient has been away from the nursing unit for 2 hours, missing the lunch delivery while awaiting a chest x-ray. To prevent hypoglycemia, the best action by the nurse is to a. save the lunch tray for the patient's later return to the unit. b. ask that diagnostic testing area staff to start a 5% dextrose IV. c. send a glass of milk or orange juice to the patient in the diagnostic testing area. d. request that if testing is further delayed, the patient be returned to the unit to eat.

ANS: D Consistency for mealtimes assists with regulation of blood glucose, so the best option is for the patient to have lunch at the usual time. Waiting to eat until after the procedure is likely to cause hypoglycemia. Administration of an IV solution is unnecessarily invasive for the patient. A glass of milk or juice will keep the patient from becoming hypoglycemic but will cause a rapid rise in blood glucose because of the rapid absorption of the simple carbohydrate in these items.

2. Which statement by a 50-yr-old female patient indicates to the nurse that further assessment of thyroid function may be necessary? a. "I notice my breasts are tender lately." b. "I am so thirsty that I drink all day long." c. "I get up several times at night to urinate." d. "I feel a lump in my throat when I swallow."

ANS: D Difficulty in swallowing can occur with a goiter. Nocturia is associated with diseases such as diabetes mellitus, diabetes insipidus, or chronic kidney disease. Breast tenderness would occur with excessive gonadal hormone levels. Thirst is a sign of disease such as diabetes.

An 18-yr-old male patient with small stature is scheduled for a growth hormone stimulation test. In preparation for the test, the nurse will obtain a. ice in a basin. c. a cardiac monitor. b. glargine insulin. d. 50% dextrose solution.

ANS: D Hypoglycemia is induced during the growth hormone stimulation test, and the nurse should be ready to administer 50% dextrose immediately. Regular insulin is used to induce hypoglycemia (glargine is never given IV). The patient does not require cardiac monitoring during the test. Although blood samples for some tests must be kept on ice, this is not true for the growth hormone stimulation test.

Which finding by the nurse when assessing a patient with a large pituitary adenoma is most important to report to the health care provider? a. Changes in visual field c. Blood glucose 150 mg/dL b. Milk leaking from breasts d. Nausea and projectile vomiting

ANS: D Nausea and projectile vomiting may indicate increased intracranial pressure, which will require rapid actions for diagnosis and treatment. Changes in the visual field, elevated blood glucose, and galactorrhea are common with pituitary adenoma, but these do not require rapid action to prevent life-threatening complications.

6. A 61-yr-old female patient admitted with pneumonia has a total serum calcium level of 13.3 mg/dL (3.3 mmol/L). The nurse will anticipate the need to teach the patient about testing for _____ levels. a. calcitonin c. thyroid hormone b. catecholamine d. parathyroid hormone

ANS: D Parathyroid hormone (PTH) is the major controller of blood calcium levels. Although calcitonin secretion is a countermechanism to PTH, it does not play a major role in calcium balance. Catecholamine and thyroid hormone levels do not affect serum calcium level.

The nurse is interviewing a new patient with diabetes who takes rosiglitazone (Avandia). Which information would the nurse anticipate resulting in the health care provider discontinuing the medication? a. The patient's blood pressure is 154/92. b. The patient's blood glucose is 86 mg/dL. c. The patient reports a history of emphysema. d. The patient has chest pressure when walking.

ANS: D Rosiglitazone can cause myocardial ischemia. The nurse should immediately notify the health care provider and expect orders to discontinue the medication. A blood glucose level of 86 mg/dL indicates a positive effect from the medication. Hypertension and a history of emphysema do not contraindicate this medication.

Which finding indicates a need to contact the health care provider before the nurse administers metformin (Glucophage)? a. The patient's blood glucose level is 174 mg/dL. b. The patient is scheduled for a chest x-ray in an hour. c. The patient has gained 2 lb (0.9 kg) in the past 24 hours. d. The patient's blood urea nitrogen (BUN) level is 52 mg/dL.

ANS: D The BUN indicates possible renal failure, and metformin should not be used in patients with renal failure. The other findings are not contraindications to the use of metformin.

Which assessment finding of a 42-yr-old patient who had a bilateral adrenalectomy requires the most rapid action by the nurse? a. The blood glucose is 192 mg/dL. b. The lungs have bibasilar crackles. c. The patient reports 6/10 incisional pain. d. The blood pressure (BP) is 88/50 mm Hg.

ANS: D The decreased BP indicates possible adrenal insufficiency. The nurse should immediately notify the health care provider so that corticosteroid medications can be administered. The nurse should also address the elevated glucose, incisional pain, and crackles with appropriate collaborative or nursing actions, but prevention and treatment of acute adrenal insufficiency are the priorities after adrenalectomy.

The nurse has been teaching a patient with type 2 diabetes about managing blood glucose levels and taking glipizide (Glucotrol). Which patient statement indicates a need for additional teaching? a. "If I overeat at a meal, I will still take the usual dose of medication." b. "Other medications besides the Glucotrol may affect my blood sugar." c. "When I am ill, I may have to take insulin to control my blood sugar." d. "My diabetes won't cause complications because I don't need insulin."

ANS: D The patient should understand that type 2 diabetes places the patient at risk for many complications and that good glucose control is as important when taking oral agents as when using insulin. The other statements are accurate and indicate good understanding of the use of glipizide.

The nurse is assigned to the care of a patient diagnosed with type 2 diabetes. In formulating a teaching plan that encourages the patient to actively participate in management of the diabetes, what should be the nurse's initial intervention? Assess patient's perception of what it means to have diabetes. Ask the patient to write down current knowledge about diabetes. Set goals for the patient to actively participate in managing his diabetes. Assume responsibility for all of the patient's care to decrease stress level.

Assess patient's perception of what it means to have diabetes. For teaching to be effective, the first step is to assess the patient. Teaching can be individualized after the nurse is aware of what a diagnosis of diabetes means to the patient. After the initial assessment, current knowledge can be assessed, and goals should be set with the patient. Assuming responsibility for all of the patient's care will not facilitate the patient's health.

A client with a diagnosis of diabetic ketoacidosis (DKA) is being treated in the emergency department. Which findings support this diagnosis? Select all that apply. Increase in pH Comatose state Deep, rapid breathing Decreased urine output Elevated blood glucose level

Comatose state Deep, rapid breathing Elevated blood glucose level Because of the profound deficiency of insulin associated with DKA, glucose cannot be used for energy and the body breaks down fat as a secondary source of energy. Ketones, which are acid byproducts of fat metabolism, build up and the client experiences a metabolic ketoacidosis. High serum glucose contributes to an osmotic diuresis and the client becomes severely dehydrated. If untreated, the client will become comatose due to severe dehydration, acidosis, and electrolyte imbalance. Kussmaul's respirations, the deep rapid breathing associated with DKA, is a compensatory mechanism by the body. The body attempts to correct the acidotic state by blowing off carbon dioxide (CO2), which is an acid. In the absence of insulin, the client will experience severe hyperglycemia. Option 1 is incorrect because in acidosis the pH would be low. Option 4 is incorrect because a high serum glucose will result in an osmotic diuresis and the client will experience polyuria.

Polydipsia and polyuria related to diabetes mellitus are primarily due to a. the release of ketones from cells during fat metabolism. b. fluid shifts resulting from the osmotic effect of hyperglycemia. c. damage to the kidneys from exposure to high levels of glucose. d. changes in RBCs resulting from attachment of excessive glucose to hemoglobin.

Correct answer: b Rationale: The osmotic effect of glucose produces the manifestations of polydipsia and polyuria.

A client with diabetes mellitus demonstrates acute anxiety when admitted to the hospital for the treatment of hyperglycemia. What is the appropriate intervention to decrease the client's anxiety? Administer a sedative. Convey empathy, trust, and respect toward the client. Ignore the signs and symptoms of anxiety, anticipating that they will soon disappear. Make sure that the client is familiar with the correct medical terms to promote understanding of what is happening.

Convey empathy, trust, and respect toward the client. Anxiety is a subjective feeling of apprehension, uneasiness, or dread. The appropriate intervention is to address the client's feelings related to the anxiety. Administering a sedative is not the most appropriate intervention and does not address the source of the client's anxiety. The nurse should not ignore the client's anxious feelings. Anxiety needs to be managed before meaningful client education can occur.

After a hypophysectomy for acromegaly, immediate postoperative nursing care should focus on a. frequent monitoring of serum and urine osmolarity. b. parenteral administration of a GH-receptor antagonist. c. keeping the patient in a recumbent position at all times. d. patient teaching regarding the need for lifelong hormone therapy.

Correct answer: a Rationale: A possible postoperative complication after a hypophysectomy is transient diabetes insipidus (DI). It may occur because of the loss of antidiuretic hormone (ADH), which is stored in the posterior lobe of the pituitary gland, or because of cerebral edema related to manipulation of the pituitary gland during surgery. To assess for DI, urine output and serum and urine osmolarity should be monitored closely.

Endocrine disorders often go unrecognized in the older adult because a. symptoms are often attributed to aging. b. older adults rarely have identifiable symptoms. c. endocrine disorders are relatively rare in the older adult. d. older adults usually have subclinical endocrine disorders that minimize symptoms.

Correct answer: a Rationale: Assessment of the effects of aging on the endocrine system is difficult because the subtle changes of aging often mimic manifestations of endocrine disorders.

To control the side effects of corticosteroid therapy, the nurse teaches the patient who is taking corticosteroids to a. increase calcium intake to 1500 mg/day. b. perform glucose monitoring for hypoglycemia. c. obtain immunizations due to high risk of infections. d. avoid abrupt position changes because of orthostatic hypotension.

Correct answer: a Rationale: Because patients often receive corticosteroid treatment for prolonged periods (more than 3 months), corticosteroid-induced osteoporosis is an important concern. Therapies to reduce the resorption of bone may include increased calcium intake, vitamin D supplementation, bisphosphonates (e.g., alendronate), and institution of a low-impact exercise program.

Which statement by the patient with type 2 diabetes is accurate? a. "I will limit my alcohol intake to one drink." b. "I am not allowed to eat any sweets because of my diabetes." c. "I cannot exercise because I take a blood glucose-lowering medication." d. "The amount of fat in my diet is not important. Only carbohydrates raise my blood sugar."

Correct answer: a Rationale: Consuming alcohol with food can reduce the risk of hypoglycemia.

Analyze the following diagnostic findings for your patient with type 2 diabetes. Which result will need further assessment? a. A1C 9% b. BP 126/80 mm Hg c. FBG 130 mg/dL (7.2 mmol/L) d. LDL cholesterol 100 mg/dL (2.6 mmol/L)

Correct answer: a Rationale: Lowering hemoglobin A1C (to less than 7%) reduces microvascular and neuropathic complications. Keeping blood glucose levels in a tighter range (normal hemoglobin A1C level, less than 6%) may further reduce complications but increases hypoglycemia risk.

A patient has a total serum calcium level of 3 mg/dL (1.5 mEq/L). If this finding reflects hypoparathyroidism, the nurse would expect further diagnostic testing to reveal a. decreased serum PTH. b. increased serum ACTH. c. increased serum glucose. d. decreased serum cortisol levels.

Correct answer: a Rationale: Normal serum calcium levels are 8.6 to 10.2 mg/dL. In hypoparathyroidism, the serum calcium level is low with a decreased serum parathyroid hormone (PTH) level.

A patient has a serum sodium level of 152 mEq/L (152 mmol/L). The normal hormonal response to this situation is a. release of ADH. b. release of ACTH. c. secretion of aldosterone. d. secretion of corticotropin-releasing hormone.

Correct answer: a Rationale: The most important stimulus of antidiuretic hormone (ADH) secretion is plasma osmolality, which is a measure of solute concentration in circulating blood. Plasma osmolality increases when there is a decrease in extracellular fluid or an increase in solute concentration. The increased plasma osmolality activates osmoreceptors, which are extremely sensitive, specialized neurons in the hypothalamus. These activated osmoreceptors stimulate ADH release. With ADH release, the renal tubules reabsorb water, which causes urine to be more concentrated.

When obtaining subjective data from a patient during assessment of the endocrine system, the nurse asks specifically about a. energy level. b. intake of vitamin C. c. employment history. d. frequency of sexual intercourse.

Correct answer: a Rationale: The nurse should ask about energy levels, particularly in comparison with the patient's past energy level. Fatigue and hyperactivity are two common problems associated with endocrine problems.

A characteristic common to all hormones is that they a. circulate in the blood bound to plasma proteins. b. influence cellular activity of specific target tissues. c. accelerate the metabolic processes of all body cells. d. enter a cell to alter the cell's metabolism or gene expression.

Correct answer: b Rationale: A hormone is a chemical substance synthesized and secreted by a specific organ or tissue. Most hormones have common characteristics, including (1) secretion in small amounts at variable but predictable rates, (2) circulation through the blood, and (3) binding to specific cell receptors in the cell membrane or within the cell.

The health care provider prescribes levothyroxine for a patient with hypothyroidism. After teaching regarding this drug, the nurse determines that further instruction is needed when the patient says a. "I can expect the medication dose may need to be adjusted." b. "I only need to take this drug until my symptoms are improved." c. "I can expect to return to normal function with the use of this drug." d. "I will report any chest pain or difficulty breathing to the doctor right away."

Correct answer: b Rationale: Levothyroxine is the drug of choice to treat hypothyroidism. The need for thyroid replacement therapy is usually lifelong.

The nurse teaches the patient that the best time to take corticosteroids for replacement purposes is a. once a day at bedtime. b. every other day on awakening. c. on arising and in the late afternoon. d. at consistent intervals every 6 to 8 hours.

Correct answer: c Rationale: As replacement therapy, glucocorticoids are usually administered in divided doses: two thirds in the morning and one third in the afternoon. This dosage schedule reflects normal circadian rhythm in endogenous hormone secretion and decreases the side effects associated with corticosteroid replacement therapy.

An important preoperative nursing intervention before an adrenalectomy for hyperaldosteronism is to a. monitor blood glucose levels. b. restrict fluid and sodium intake. c. administer potassium-sparing diuretics. d. advise the patient to make postural changes slowly.

Correct answer: c Rationale: Before surgery, patients should be treated with potassium-sparing diuretics (spironolactone, eplerenone) to normalize serum potassium levels. Spironolactone and eplerenone block the binding of aldosterone to the mineralocorticoid receptor in the terminal distal tubules and collecting ducts of the kidney, thus increasing sodium excretion, water excretion, and potassium retention. Oral potassium supplements may also be necessary.

A patient with a head injury develops SIADH. Manifestations the nurse would expect to find include a. hypernatremia and edema. b. muscle spasticity and hypertension. c. low urine output and hyponatremia. d. weight gain and decreased glomerular filtration rate.

Correct answer: c Rationale: Excess ADH increases the permeability of the renal distal tubule and collecting ducts, which leads to the reabsorption of water into the circulation. Consequently, extracellular fluid volume expands, plasma osmolality declines, the glomerular filtration rate increases, and sodium levels decline (i.e., dilutional hyponatremia). Hyponatremia causes muscle cramping, pain, and weakness. Initially, the patient displays thirst, dyspnea on exertion, and fatigue. Patients with the syndrome of inappropriate antidiuretic hormone secretion (SIADH) experience low urinary output and increased body weight. As the serum sodium level falls (usually to less than 120 mEq/L), manifestations become more severe and include headache, vomiting, abdominal cramps, muscle twitching, and seizures. As plasma osmolality and serum sodium levels continue to decline, cerebral edema may occur, leading to lethargy, anorexia, confusion, seizures, and coma.

A patient with diabetes has a serum glucose level of 824 mg/dL (45.7 mmol/L) and is unresponsive. After assessing the patient, the nurse suspects diabetic ketoacidosis rather than hyperosmolar hyperglycemic syndrome based on the finding of a. polyuria. b. severe dehydration. c. rapid, deep respirations. d. decreased serum potassium.

Correct answer: c Rationale: Signs and symptoms of DKA include manifestations of dehydration, such as poor skin turgor, dry mucous membranes, tachycardia, and orthostatic hypotension. Early symptoms may include lethargy and weakness. As the patient becomes severely dehydrated, the skin becomes dry and loose, and the eyeballs become soft and sunken. Abdominal pain is another symptom of DKA that may be accompanied by anorexia and vomiting. Kussmaul respirations (i.e., rapid, deep breathing associated with dyspnea) are the body's attempt to reverse metabolic acidosis through the exhalation of excess carbon dioxide. Acetone is identified on the breath as a sweet, fruity odor. Laboratory findings include a blood glucose level greater than 250 mg/dL, arterial blood pH less than 7.30, serum bicarbonate level less than 15 mEq/L, and moderate to high ketone levels in the urine or blood.

A patient is receiving radiation therapy for cancer of the kidney. The nurse monitors the patient for signs and symptoms of damage to the a. pancreas. b. thyroid gland. c. adrenal glands. d. posterior pituitary gland

Correct answer: c Rationale: The adrenal glands are small, paired, highly vascularized glands located on the upper portion of each kidney. These glands may be damaged by radiation of the kidneys.

An appropriate technique to use during physical assessment of the thyroid gland is a. asking the patient to hyperextend the neck during palpation. b. percussing the neck for dullness to define the size of the thyroid. c. having the patient swallow water during inspection and palpation of the gland. d. using deep palpation to determine the extent of a visibly enlarged thyroid gland.

Correct answer: c Rationale: Water should always be available for the patient to swallow as part of inspection and palpation of the thyroid gland.

Which statement would be correct for a patient with type 2 diabetes who was admitted to the hospital with pneumonia? a. The patient must receive insulin therapy to prevent ketoacidosis. b. The patient has islet cell antibodies that have destroyed the pancreas's ability to produce insulin. c. The patient has minimal or absent endogenous insulin secretion and requires daily insulin injections. d. The patient may have sufficient endogenous insulin to prevent ketosis but is at risk for hyperosmolar hyperglycemic syndrome.

Correct answer: d Rationale: Hyperosmolar hyperglycemic syndrome (HHS) is a life-threatening syndrome that can occur in a patient with diabetes who is able to produce enough insulin to prevent diabetic ketoacidosis (DKA) but not enough to prevent severe hyperglycemia, osmotic diuresis, and extracellular fluid depletion.

Which are appropriate therapies for patients with diabetes mellitus (select all that apply)? a. Use of statins to reduce CVD risk b. Use of diuretics to treat nephropathy c. Use of ACE inhibitors to treat nephropathy d. Use of serotonin agonists to decrease appetite

Correct answers: a, c, e Rationale: In patients with diabetes who have albuminuria, angiotensin-converting enzyme (ACE) inhibitors (e.g., lisinopril [Prinivil, Zestril]) or angiotensin II receptor antagonists (ARBs) (e.g., losartan [Cozaar]) are used. Both classes of drugs are used to treat hypertension and have been found to delay the progression of nephropathy in patients with diabetes. The statin drugs are the most widely used lipid-lowering agents. Laser photocoagulation therapy is indicated to reduce the risk of vision loss in patients with proliferative retinopathy, in those with macular edema, and in some cases of nonproliferative retinopathy.

An abnormal finding by the nurse during an endocrine assessment would be (select all that apply) a. blood pressure of 100/70 mm Hg. b. excessive facial hair on a woman. c. soft, formed stool every other day. d. 3-lb weight gain over last 6 months. e. hyperpigmented coloration in lower legs.

Correct answers: b, e Rationale: Hirsutism (i.e., excessive facial hair on women) may indicate Cushing syndrome or prolactinoma, a pituitary tumor. Hyperpigmentation (i.e., darkening of the skin, particularly in creases and skinfolds) may indicate Addison's disease, which is associated with increased secretion of melanocyte-stimulating hormone, or it may indicate acanthosis nigricans.

Important nursing intervention(s) when caring for a patient with Cushing syndrome include (select all that apply) a. restricting protein intake. b. monitoring blood glucose levels. c. observing for signs of hypotension. d. administering medication in equal doses. e. protecting patient from exposure to infection.

Correct answers: b, e Rationale: Hyperglycemia occurs with Cushing disease because of glucose intolerance (associated with cortisol-induced insulin resistance) and increased gluconeogenesis by the liver. High levels of corticosteroids increase susceptibility to infection and delay wound healing.

What should be included in the interprofessional plan of care for a patient with Cushing disease? Lab monitoring for hyperkalemia Vital sign monitoring for hypotension Counseling related to body image changes Diet consultation to determine low protein choices

Counseling related to body image changes Elevated corticosteroid levels can cause body changes, including truncal obesity, moon face, and hirsutism in women and gynecomastia in men. Counseling and support should be offered because of the changes in body image. Hypokalemia and hypertension are consistent with Cushing disease. Sodium restriction and potassium supplementation are indicated. High protein choices are necessary to counteract catabolic processes and assist with wound healing

The nurse is caring for a patient receiving high-dose oral corticosteroid therapy after a kidney transplant. Which potential side effect presents the greatest risk? Infection Low blood pressure Increased urine output Decreased blood glucose

Infection Side effects of corticosteroid therapy include increased susceptibility to infection, edema related to sodium and water retention (decreasing urine output), hypertension, and hyperglycemia.

A client is brought to the emergency department in an unresponsive state, and a diagnosis of hyperosmolar hyperglycemic syndrome is made. The nurse would immediately prepare to initiate which anticipated health care provider's prescription? Endotracheal intubation 100 units of NPH insulin Intravenous infusion of normal saline Intravenous infusion of sodium bicarbonate

Intravenous infusion of normal saline The primary goal of treatment in hyperosmolar hyperglycemic syndrome (HHS) is to rehydrate the client to restore fluid volume and to correct electrolyte deficiency. Intravenous (IV) fluid replacement is similar to that administered in diabetic ketoacidosis (DKA) and begins with IV infusion of normal saline. Regular insulin, not NPH insulin, would be administered. The use of sodium bicarbonate to correct acidosis is avoided because it can precipitate a further drop in serum potassium levels. Intubation and mechanical ventilation are not required to treat HHS.

An external insulin pump is prescribed for a client with diabetes mellitus. When the client asks the nurse about the functioning of the pump, the nurse bases the response on which information about the pump? It is timed to release programmed doses of either short-duration or NPH insulin into the bloodstream at specific intervals. It continuously infuses small amounts of NPH insulin into the bloodstream while regularly monitoring blood glucose levels. It is surgically attached to the pancreas and infuses regular insulin into the pancreas. This releases insulin into the bloodstream. It administers a small continuous dose of short-duration insulin subcutaneously. The client can self-administer an additional bolus dose from the pump before each meal.

It administers a small continuous dose of short-duration insulin subcutaneously. The client can self-administer an additional bolus dose from the pump before each meal. An insulin pump provides a small continuous dose of short-duration (rapid- or short-acting) insulin subcutaneously throughout the day and night. The client can self-administer an additional bolus dose from the pump before each meal as needed. Short-duration insulin is used in an insulin pump. An external pump is not attached surgically to the pancreas.

The patient with systemic lupus erythematosus is diagnosed with syndrome of inappropriate antidiuretic hormone (SIADH). What should be included in the plan of care (select all that apply.)? Obtain weekly weights. Limit fluids to 1000 mL/day. Monitor for signs of hypernatremia. Administration of diuretics as ordered. Minimize turning and range of motion. Keep the head of the bed at 10 degrees or less elevation.

Limit fluids to 1000 mL/day. Administration of diuretics as ordered. Keep the head of the bed at 10 degrees or less elevation. The care for the patient with SIADH will include limiting fluids to 1000 mL/day or less to decrease weight, increase osmolality, and improve symptoms and keeping the head of the bed elevated at 10 degrees or less to enhance venous return to the heart and increase left atrial filling pressure, thereby reducing the release of ADH. Measure weights daily and maintain accurate intake and output. Monitor for signs of hyponatremia. Frequent turning, positioning, and range-of-motion exercises are important to maintain skin integrity and joint mobility.

The nurse is teaching a patient who has diabetes about vascular complications of diabetes. What information is appropriate for the nurse to include? Macroangiopathy does not occur in type 1 diabetes but rather in type 2 diabetics who have severe disease. Microangiopathy is specific to diabetes and most commonly affects the capillary membranes of the eyes, kidneys, and skin. Renal damage resulting from changes in large- and medium-sized blood vessels can be prevented by careful glucose control. Macroangiopathy causes slowed gastric emptying and the sexual impotency experienced by a majority of patients with diabetes.

Microangiopathy is specific to diabetes and most commonly affects the capillary membranes of the eyes, kidneys, and skin. Microangiopathy occurs in diabetes mellitus. When it affects the eyes, it is called diabetic retinopathy. When the kidneys are affected, the patient has nephropathy. When the skin is affected, it can lead to diabetic foot ulcers. Macroangiopathy can occur in either type 1 or type 2 diabetes and contributes to cerebrovascular, cardiovascular, and peripheral vascular disease. Sexual impotency and slowed gastric emptying result from microangiopathy and neuropathy.

The nurse is caring for a patient after a thyroidectomy. Which assessment findings would indicate the presence of possible hypoparathyroidism? Polyuria, polydipsia, and weight loss Cardiac dysrhythmias and hypertension Muscle spasms and hyperactive deep tendon reflexes Hyperpigmentation, skin ulcers, and peripheral edema

Muscle spasms and hyperactive deep tendon reflexes Common assessment abnormalities associated with hypoparathyroidism include tetany (muscle spasms) and increased deep tendon reflexes. Hyperpigmentation is associated with Addison's disease. Skin ulcers occur in patients with diabetes. Edema is associated with hypothyroidism. Polyuria and polydipsia occur in patients with diabetes mellitus or diabetes insipidus. Weight loss occurs in hyperthyroidism or diabetic ketoacidosis. Hypertension and cardiac dysrhythmias may result from hyperthyroidism, hyperparathyroidism, or pheochromocytoma.

The nurse is caring for a patient admitted with suspected hyperparathyroidism. Which signs and symptoms would represent the expected electrolyte imbalance (select all that apply.)? Nausea and vomiting Neurologic irritability Lethargy and weakness Increasing urine output Hyperactive bowel sounds

Nausea and vomiting Lethargy and weakness Increasing urine output Hyperparathyroidism can cause hypercalcemia. Signs of hypercalcemia include muscle weakness, polyuria, constipation, nausea and vomiting, lethargy, and memory impairment. Neurologic irritability and hyperactive bowel sounds do not occur with hypercalcemia.

A patient who smokes reports having significant stress and is experiencing eye problems. On assessment, the nurse notes exophthalmos. What additional abnormal findings should the nurse assess for? Muscle weakness and slow movements Puffy face, decreased sweating, and dry hair Systolic hypertension and increased heart rate Decreased appetite, increased thirst, and pallor

Systolic hypertension and increased heart rate The patient's manifestations are consistent with Graves' disease or hyperthyroidism. Systolic hypertension, increased heart rate, and increased thirst are associated with hyperthyroidism. Cigarette smoking places the patient at increased risk of developing Graves' disease. The inhaled cigarette toxins may absorb via the eye orbits, causing exophthalmos. A puffy face; decreased sweating; dry, coarse hair; muscle weakness and slow movements; decreased appetite; and pallor are all manifestations of hypothyroidism.

When teaching a patient about a urine study for free cortisol, what is most important for the nurse to tell the patient? Save the first voided urine in the morning. Maintain a high-sodium diet 3 days before collection. Try to avoid stressful situations during the collection period. Complete at least 30 minutes of exercise before collecting the urine sample.

Try to avoid stressful situations during the collection period. A urine study for free cortisol requires a 24-hour urine collection. Instruct the patient to avoid stressful situations and excessive physical exercise because these could unduly increase cortisol levels. The patient should maintain a low-sodium diet before and during the urine collection period.


Ensembles d'études connexes

Classifications of Consequences- Concepts and Functions of ABA

View Set

Physics: Free fall and Projectile Motion

View Set

Trading Markets: Trading Markets Basics

View Set

BA Chapter 10: designing organization structure

View Set

Operating System Overview (Quiz 1)

View Set